Programs & Examples On #Popupwindow

A window that appears ('pops up', hence the name) when you select an option with a mouse or press a designated function key. Usually, the pop-up window contains a menu of commands and stays on the screen only until you select one of the commands. It then disappears.

how to make window.open pop up Modal?

You can't make window.open modal and I strongly recommend you not to go that way. Instead you can use something like jQuery UI's dialog widget.

UPDATE:

You can use load() method:

$("#dialog").load("resource.php").dialog({options});

This way it would be faster but the markup will merge into your main document so any submit will be applied on the main window.

And you can use an IFRAME:

$("#dialog").append($("<iframe></iframe>").attr("src", "resource.php")).dialog({options});

This is slower, but will submit independently.

How to create a popup window (PopupWindow) in Android

Edit your style.xml with:

<style name="AppTheme" parent="Base.V21.Theme.AppCompat.Light.Dialog">

Base.V21.Theme.AppCompat.Light.Dialog provides a android poup-up theme

Popup window in PHP?

if (isset($_POST['Register']))
    {
        $ErrorArrays = array (); //Empty array for input errors 

        $Input_Username = $_POST['Username'];
        $Input_Password = $_POST['Password'];
        $Input_Confirm = $_POST['ConfirmPass'];
        $Input_Email = $_POST['Email'];

        if (empty($Input_Username))
        {
            $ErrorArrays[] = "Username Is Empty";
        }
        if (empty($Input_Password))
        {
            $ErrorArrays[] = "Password Is Empty";
        }
        if ($Input_Password !== $Input_Confirm)
        {
            $ErrorArrays[] = "Passwords Do Not Match!";
        }
        if (!filter_var($Input_Email, FILTER_VALIDATE_EMAIL))
        {
            $ErrorArrays[] = "Incorrect Email Formatting";
        }

        if (count($ErrorArrays) == 0)
        {
            // No Errors
        }
        else
        {
            foreach ($ErrorArrays AS $Errors)
            {
                echo "<font color='red'><b>".$Errors."</font></b><br>";
            }
        }
    }

?>

    <form method="POST"> 
        Username: <input type='text' name='Username'> <br>
        Password: <input type='password' name='Password'><br>
        Confirm Password: <input type='password' name='ConfirmPass'><br>
        Email: <input type='text' name='Email'> <br><br>

        <input type='submit' name='Register' value='Register'>


    </form> 

This is a very basic PHP Form validation. This could be put in a try block, but for basic reference, I see this fit following our conversation in the comment box.

What this script will do, is process each of the post elements, and act accordingly, for example:

    if (!filter_var($Input_Email, FILTER_VALIDATE_EMAIL))
        {
            $ErrorArrays[] = "Incorrect Email Formatting";
        }

This will check:

if $Input_Email is not a valid email. If this is not a valid E-mail, then a message will get added to a empty array.

Further down the script, you will see:

    if (count($ErrorArrays) == 0)
    {
        // No Errors
    }
    else
    {
        foreach ($ErrorArrays AS $Errors)
        {
            echo "<font color='red'><b>".$Errors."</font></b><br>";
        }
    }

Basically. if the array count is not 0, errors have been found. Then the script will print out the errors.

Remember, this is a reference based on our conversation in the comment box, and should be used as such.

How to handle Pop-up in Selenium WebDriver using Java

You can handle popup window or alert box:

Alert alert = driver.switchTo().alert();
alert.accept();

You can also decline the alert box:

Alert alert = driver.switchTo().alert();
alert().dismiss();

Blur or dim background when Android PopupWindow active

For me, something like Abdelhak Mouaamou's answer works, tested on API level 16 and 27.

Instead of using popupWindow.getContentView().getParent() and casting the result to View (which crashes on API level 16 cause there it returns a ViewRootImpl object which isn't an instance of View) I just use .getRootView() which returns a view already, so no casting required there.

Hope it helps someone :)

complete working example scrambled together from other stackoverflow posts, just copy-paste it, e.g., in the onClick listener of a button:

// inflate the layout of the popup window
LayoutInflater inflater = (LayoutInflater)getSystemService(LAYOUT_INFLATER_SERVICE);
if(inflater == null) {
    return;
}
//View popupView = inflater.inflate(R.layout.my_popup_layout, null); // this version gives a warning cause it doesn't like null as argument for the viewRoot, c.f. https://stackoverflow.com/questions/24832497 and https://stackoverflow.com/questions/26404951
View popupView = View.inflate(MyParentActivity.this, R.layout.my_popup_layout, null);

// create the popup window
final PopupWindow popupWindow = new PopupWindow(popupView,
        LinearLayout.LayoutParams.WRAP_CONTENT,
        LinearLayout.LayoutParams.WRAP_CONTENT,
        true // lets taps outside the popup also dismiss it
        );

// do something with the stuff in your popup layout, e.g.:
//((TextView)popupView.findViewById(R.id.textview_popup_helloworld))
//      .setText("hello stackoverflow");

// dismiss the popup window when touched
popupView.setOnTouchListener(new View.OnTouchListener() {
        @Override
        public boolean onTouch(View v, MotionEvent event) {
            popupWindow.dismiss();
            return true;
        }
});

// show the popup window
// which view you pass in doesn't matter, it is only used for the window token
popupWindow.showAtLocation(view, Gravity.CENTER, 0, 0);
//popupWindow.setOutsideTouchable(false); // doesn't seem to change anything for me

View container = popupWindow.getContentView().getRootView();
if(container != null) {
    WindowManager wm = (WindowManager)getSystemService(Context.WINDOW_SERVICE);
    WindowManager.LayoutParams p = (WindowManager.LayoutParams)container.getLayoutParams();
    p.flags = WindowManager.LayoutParams.FLAG_DIM_BEHIND;
    p.dimAmount = 0.3f;
    if(wm != null) {
        wm.updateViewLayout(container, p);
    }
}

RegisterStartupScript from code behind not working when Update Panel is used

You need to use ScriptManager.RegisterStartupScript for Ajax.

protected void ButtonPP_Click(object sender, EventArgs e) {     if (radioBtnACO.SelectedIndex < 0)     {         string csname1 = "PopupScript";          var cstext1 = new StringBuilder();         cstext1.Append("alert('Please Select Criteria!')");          ScriptManager.RegisterStartupScript(this, GetType(), csname1,             cstext1.ToString(), true);     } } 

Add a UIView above all, even the navigation bar

Swift version of @Nicolas Bonnet 's answer:

    var popupWindow: UIWindow?

func showViewController(controller: UIViewController) {
    self.popupWindow = UIWindow(frame: UIScreen.mainScreen().bounds)
    controller.view.frame = self.popupWindow!.bounds
    self.popupWindow!.rootViewController = controller
    self.popupWindow!.makeKeyAndVisible()
}

func viewControllerDidRemove() {
    self.popupWindow?.removeFromSuperview()
    self.popupWindow = nil
}

Don't forget that the window must be a strong property, because the original answer leads to an immediate deallocation of the window

jQuery function to open link in new window

Try this,

$('.popup').click(function(event) {
    event.preventDefault();
    window.open($(this).attr("href"), "popupWindow", "width=600,height=600,scrollbars=yes");
});

You have to include jQuery reference to work this, here is the working sampe http://jsfiddle.net/a7qJt/

How do I add a Fragment to an Activity with a programmatically created content view

public abstract class SingleFragmentActivity extends Activity {

    public static final String FRAGMENT_TAG = "single";
    private Fragment fragment;

    @Override
    protected void onCreate(Bundle savedInstanceState) {
    super.onCreate(savedInstanceState);
        if (savedInstanceState == null) {
            fragment = onCreateFragment();
           getFragmentManager().beginTransaction()
                   .add(android.R.id.content, fragment, FRAGMENT_TAG)
                   .commit();
       } else {
           fragment = getFragmentManager().findFragmentByTag(FRAGMENT_TAG);
       }
   }

   public abstract Fragment onCreateFragment();

   public Fragment getFragment() {
       return fragment;
   }

}

use

public class ViewCatalogItemActivity extends SingleFragmentActivity {
    @Override
    public Fragment onCreateFragment() {
        return new FragmentWorkShops();
    }

}

Bootstrap 4, how to make a col have a height of 100%?

Set display: table for parent div and display: table-cell for children divs

HTML :

<div class="container-fluid">
  <div class="row justify-content-center display-as-table">

    <div class="col-4 hidden-md-down" id="yellow">
      XXXX<br />
      XXXX<br />
      XXXX<br />
      XXXX<br />
      XXXX<br />
      XXXX<br />vv
      XXXX<br />
    </div>

    <div class="col-10 col-sm-10 col-md-10 col-lg-8 col-xl-8" id="red">
      Form Goes Here
    </div>
  </div>
</div>

CSS:

#yellow {
  height: 100%;
  background: yellow;
  width: 50%;
}
#red {background: red}

.container-fluid {bacgkround: #ccc}

/* this is the part make equal height */
.display-as-table {display: table; width: 100%;}
.display-as-table > div {display: table-cell; float: none;}

Calling a php function by onclick event

The onClick attribute of html tags only takes Javascript but not PHP code. However, you can easily call a PHP function from within the Javascript code by using the JS document.write() function - effectively calling the PHP function by "writing" its call to the browser window: Eg.

onclick="document.write('<?php //call a PHP function here ?>');"

Your example:

    <?php
          function hello(){
              echo "Hello";
          }
    ?>

<input type="button" name="Release" onclick="document.write('<?php hello() ?>');" value="Click to Release">

How to convert SecureString to System.String?

I think it would be best for SecureString dependent functions to encapsulate their dependent logic in an anonymous function for better control over the decrypted string in memory (once pinned).

The implementation for decrypting SecureStrings in this snippet will:

  1. Pin the string in memory (which is what you want to do but appears to be missing from most answers here).
  2. Pass its reference to the Func/Action delegate.
  3. Scrub it from memory and release the GC in the finally block.

This obviously makes it a lot easier to "standardize" and maintain callers vs. relying on less desirable alternatives:

  • Returning the decrypted string from a string DecryptSecureString(...) helper function.
  • Duplicating this code wherever it is needed.

Notice here, you have two options:

  1. static T DecryptSecureString<T> which allows you to access the result of the Func delegate from the caller (as shown in the DecryptSecureStringWithFunc test method).
  2. static void DecryptSecureString is simply a "void" version which employ an Action delegate in cases where you actually don't want/need to return anything (as demonstrated in the DecryptSecureStringWithAction test method).

Example usage for both can be found in the StringsTest class included.

Strings.cs

using System;
using System.Runtime.InteropServices;
using System.Security;

namespace SecurityUtils
{
    public partial class Strings
    {
        /// <summary>
        /// Passes decrypted password String pinned in memory to Func delegate scrubbed on return.
        /// </summary>
        /// <typeparam name="T">Generic type returned by Func delegate</typeparam>
        /// <param name="action">Func delegate which will receive the decrypted password pinned in memory as a String object</param>
        /// <returns>Result of Func delegate</returns>
        public static T DecryptSecureString<T>(SecureString secureString, Func<string, T> action)
        {
            var insecureStringPointer = IntPtr.Zero;
            var insecureString = String.Empty;
            var gcHandler = GCHandle.Alloc(insecureString, GCHandleType.Pinned);

            try
            {
                insecureStringPointer = Marshal.SecureStringToGlobalAllocUnicode(secureString);
                insecureString = Marshal.PtrToStringUni(insecureStringPointer);

                return action(insecureString);
            }
            finally
            {
                //clear memory immediately - don't wait for garbage collector
                fixed(char* ptr = insecureString )
                {
                    for(int i = 0; i < insecureString.Length; i++)
                    {
                        ptr[i] = '\0';
                    }
                }

                insecureString = null;

                gcHandler.Free();
                Marshal.ZeroFreeGlobalAllocUnicode(insecureStringPointer);
            }
        }

        /// <summary>
        /// Runs DecryptSecureString with support for Action to leverage void return type
        /// </summary>
        /// <param name="secureString"></param>
        /// <param name="action"></param>
        public static void DecryptSecureString(SecureString secureString, Action<string> action)
        {
            DecryptSecureString<int>(secureString, (s) =>
            {
                action(s);
                return 0;
            });
        }
    }
}

StringsTest.cs

using Microsoft.VisualStudio.TestTools.UnitTesting;
using System.Security;

namespace SecurityUtils.Test
{
    [TestClass]
    public class StringsTest
    {
        [TestMethod]
        public void DecryptSecureStringWithFunc()
        {
            // Arrange
            var secureString = new SecureString();

            foreach (var c in "UserPassword123".ToCharArray())
                secureString.AppendChar(c);

            secureString.MakeReadOnly();

            // Act
            var result = Strings.DecryptSecureString<bool>(secureString, (password) =>
            {
                return password.Equals("UserPassword123");
            });

            // Assert
            Assert.IsTrue(result);
        }

        [TestMethod]
        public void DecryptSecureStringWithAction()
        {
            // Arrange
            var secureString = new SecureString();

            foreach (var c in "UserPassword123".ToCharArray())
                secureString.AppendChar(c);

            secureString.MakeReadOnly();

            // Act
            var result = false;

            Strings.DecryptSecureString(secureString, (password) =>
            {
                result = password.Equals("UserPassword123");
            });

            // Assert
            Assert.IsTrue(result);
        }
    }
}

Obviously, this doesn't prevent abuse of this function in the following manner, so just be careful not to do this:

[TestMethod]
public void DecryptSecureStringWithAction()
{
    // Arrange
    var secureString = new SecureString();

    foreach (var c in "UserPassword123".ToCharArray())
        secureString.AppendChar(c);

    secureString.MakeReadOnly();

    // Act
    string copyPassword = null;

    Strings.DecryptSecureString(secureString, (password) =>
    {
        copyPassword = password; // Please don't do this!
    });

    // Assert
    Assert.IsNull(copyPassword); // Fails
}

Happy coding!

How to parse JSON data with jQuery / JavaScript?

I agree with all the above solutions, but to point out whats the root cause of this issue is, that major role player in all above code is this line of code:

dataType:'json'

when you miss this line (which is optional), the data returned from server is treated as full length string (which is default return type). Adding this line of code informs jQuery to convert the possible json string into json object.

Any jQuery ajax calls should specify this line, if expecting json data object.

git with development, staging and production branches

Actually what made this so confusing is that the Beanstalk people stand behind their very non-standard use of Staging (it comes before development in their diagram, and it's not a mistake!

https://twitter.com/Beanstalkapp/status/306129447885631488

ASP.NET Core form POST results in a HTTP 415 Unsupported Media Type response

In my case, I received the HTTP 415 Unsupported Media Type response, since I specified the content type to be TEXT and NOT JSON, so simply changing the type solved the issue. Please check the solution in more detail in the following blog post: https://www.howtodevelop.net/article/20/unsupported-media-type-415-in-aspnet-core-web-api

Remove files from Git commit

Another approach that we can do is to :

  1. Delete the file
  2. Make a new commit with the deleted file
  3. Do an interactive rebase and squash both commits
  4. Push

Get current application physical path within Application_Start

There's, however, slight difference among all these options which

I found out that

If you do

    string URL = Server.MapPath("~");

or

    string URL = Server.MapPath("/");

or

    string URL = HttpRuntime.AppDomainAppPath;

your URL will display resources in your link like this:

    "file:///d:/InetPUB/HOME/Index/bin/Resources/HandlerDoc.htm"

But if you want your URL to show only virtual path not the resources location, you should do

    string URL = HttpRuntime.AppDomainAppVirtualPath; 

then, your URL is displaying a virtual path to your resources as below

    "http://HOME/Index/bin/Resources/HandlerDoc.htm"        

Java - Convert integer to string

Always use either String.valueOf(number) or Integer.toString(number).

Using "" + number is an overhead and does the following:

StringBuilder sb = new StringBuilder();
sb.append("");
sb.append(number);
return sb.toString();

pandas create new column based on values from other columns / apply a function of multiple columns, row-wise

As @user3483203 pointed out, numpy.select is the best approach

Store your conditional statements and the corresponding actions in two lists

conds = [(df['eri_hispanic'] == 1),(df[['eri_afr_amer', 'eri_asian', 'eri_hawaiian', 'eri_nat_amer', 'eri_white']].sum(1).gt(1)),(df['eri_nat_amer'] == 1),(df['eri_asian'] == 1),(df['eri_afr_amer'] == 1),(df['eri_hawaiian'] == 1),(df['eri_white'] == 1,])

actions = ['Hispanic', 'Two Or More', 'A/I AK Native', 'Asian', 'Black/AA', 'Haw/Pac Isl.', 'White']

You can now use np.select using these lists as its arguments

df['label_race'] = np.select(conds,actions,default='Other')

Reference: https://numpy.org/doc/stable/reference/generated/numpy.select.html

How to mark a method as obsolete or deprecated?

With ObsoleteAttribute you can to show the deprecated method. Obsolete attribute has three constructor:

  1. [Obsolete]: is a no parameter constructor and is a default using this attribute.
  2. [Obsolete(string message)]: in this format you can get message of why this method is deprecated.
  3. [Obsolete(string message, bool error)]: in this format message is very explicit but error means, in compilation time, compiler must be showing error and cause to fail compiling or not.

enter image description here

Set CSS property in Javascript?

if you want to add a global property, you can use:

    var styleEl = document.createElement('style'), styleSheet;
            document.head.appendChild(styleEl);
            styleSheet = styleEl.sheet;
            styleSheet.insertRule(".modal { position:absolute; bottom:auto; }", 0);

Declare and assign multiple string variables at the same time

Try

string     Camnr , Klantnr , Ordernr , Bonnr , Volgnr , Omschrijving , Startdatum ,    Bonprioriteit , Matsoort , Dikte , Draaibaarheid , Draaiomschrijving , Orderleverdatum , Regeltaakkode , Gebruiksvoorkeur , Regelcamprog , Regeltijd , Orderrelease ;

and then

Camnr = Klantnr = Ordernr = Bonnr = Volgnr = Omschrijving = Startdatum = Bonprioriteit = Matsoort = Dikte = Draaibaarheid = Draaiomschrijving = Orderleverdatum = Regeltaakkode = Gebruiksvoorkeur = Regelcamprog = Regeltijd = Orderrelease = "";

Reflection: How to Invoke Method with parameters

I'am posting this answer because many visitors enter here from google for this problem.


string result = this.GetType().GetMethod("Print").Invoke(this, new object[]{"firstParam", 157, "third_Parammmm" } );

when external .dll -instead of this.GetType(), you might use typeof(YourClass).

writing integer values to a file using out.write()

Also you can use f-string formatting to write integer to file

For appending use following code, for writing once replace 'a' with 'w'.

for i in s_list:
    with open('path_to_file','a') as file:
        file.write(f'{i}\n')

file.close()

Spring MVC: difference between <context:component-scan> and <annotation-driven /> tags?

<mvc:annotation-driven /> means that you can define spring beans dependencies without actually having to specify a bunch of elements in XML or implement an interface or extend a base class. For example @Repository to tell spring that a class is a Dao without having to extend JpaDaoSupport or some other subclass of DaoSupport. Similarly @Controller tells spring that the class specified contains methods that will handle Http requests without you having to implement the Controller interface or extend a subclass that implements the controller.

When spring starts up it reads its XML configuration file and looks for <bean elements within it if it sees something like <bean class="com.example.Foo" /> and Foo was marked up with @Controller it knows that the class is a controller and treats it as such. By default, Spring assumes that all the classes it should manage are explicitly defined in the beans.XML file.

Component scanning with <context:component-scan base-package="com.mycompany.maventestwebapp" /> is telling spring that it should search the classpath for all the classes under com.mycompany.maventestweapp and look at each class to see if it has a @Controller, or @Repository, or @Service, or @Component and if it does then Spring will register the class with the bean factory as if you had typed <bean class="..." /> in the XML configuration files.

In a typical spring MVC app you will find that there are two spring configuration files, a file that configures the application context usually started with the Spring context listener.

<listener>
    <listener-class>org.springframework.web.context.ContextLoaderListener</listener-class>
</listener>

And a Spring MVC configuration file usually started with the Spring dispatcher servlet. For example.

<servlet>
        <servlet-name>main</servlet-name>
        <servlet-class>org.springframework.web.servlet.DispatcherServlet</servlet-class>
        <load-on-startup>1</load-on-startup>
    </servlet>
    <servlet-mapping>
        <servlet-name>main</servlet-name>
        <url-pattern>/</url-pattern>
    </servlet-mapping>

Spring has support for hierarchical bean factories, so in the case of the Spring MVC, the dispatcher servlet context is a child of the main application context. If the servlet context was asked for a bean called "abc" it will look in the servlet context first, if it does not find it there it will look in the parent context, which is the application context.

Common beans such as data sources, JPA configuration, business services are defined in the application context while MVC specific configuration goes not the configuration file associated with the servlet.

Hope this helps.

Is there a way to specify how many characters of a string to print out using printf()?

The basic way is:

printf ("Here are the first 8 chars: %.8s\n", "A string that is more than 8 chars");

The other, often more useful, way is:

printf ("Here are the first %d chars: %.*s\n", 8, 8, "A string that is more than 8 chars");

Here, you specify the length as an int argument to printf(), which treats the '*' in the format as a request to get the length from an argument.

You can also use the notation:

printf ("Here are the first 8 chars: %*.*s\n",
        8, 8, "A string that is more than 8 chars");

This is also analogous to the "%8.8s" notation, but again allows you to specify the minimum and maximum lengths at runtime - more realistically in a scenario like:

printf("Data: %*.*s Other info: %d\n", minlen, maxlen, string, info);

The POSIX specification for printf() defines these mechanisms.

How to set cell spacing and UICollectionView - UICollectionViewFlowLayout size ratio?

For Swift 3 and XCode 8, this worked. Follow below steps to achieve this:-

{
    let layout: UICollectionViewFlowLayout = UICollectionViewFlowLayout()
    let width = UIScreen.main.bounds.width
    layout.sectionInset = UIEdgeInsets(top: 0, left: 5, bottom: 0, right: 5)
    layout.itemSize = CGSize(width: width / 2, height: width / 2)
    layout.minimumInteritemSpacing = 0
    layout.minimumLineSpacing = 0
    collectionView!.collectionViewLayout = layout
}

Place this code into viewDidLoad() function.

PHPMailer - SMTP ERROR: Password command failed when send mail from my server

As others already suggested, you can enable the "less secure" applications or you can simply switch from ssl to tls:

$mailer->Host = 'tls://smtp.gmail.com';
$mailer->SMTPAuth = true;
$mailer->Username = "[email protected]";
$mailer->Password = "***";
$mailer->SMTPSecure = 'tls';
$mailer->Port = 587;

When using tls there's no need to grant access for less secure applications, just make sure, IMAP is enabled.

Moment Js UTC to Local Time

Here is what I do using Intl api:

let currentTimeZone = new Intl.DateTimeFormat().resolvedOptions().timeZone; // For example: Australia/Sydney

this will return a time zone name. Pass this parameter to the following function to get the time

let dateTime = new Date(date).toLocaleDateString('en-US',{ timeZone: currentTimeZone, hour12: true});

let time = new Date(date).toLocaleTimeString('en-US',{ timeZone: currentTimeZone, hour12: true});

you can also format the time with moment like this:

moment(new Date(`${dateTime} ${time}`)).format('YYYY-MM-DD[T]HH:mm:ss');

Converting List<String> to String[] in Java

You want

String[] strarray = strlist.toArray(new String[0]);

See here for the documentation and note that you can also call this method in such a way that it populates the passed array, rather than just using it to work out what type to return. Also note that maybe when you print your array you'd prefer

System.out.println(Arrays.toString(strarray));

since that will print the actual elements.

How to extract one column of a csv file

Landed here looking to extract from a tab separated file. Thought I would add.

cat textfile.tsv | cut -f2 -s

Where -f2 extracts the 2, non-zero indexed column, or the second column.

How do you find the first key in a dictionary?

So I found this page while trying to optimize a thing for taking the only key in a dictionary of known length 1 and returning only the key. The below process was the fastest for all dictionaries I tried up to size 700.

I tried 7 different approaches, and found that this one was the best, on my 2014 Macbook with Python 3.6:

def first_5():
    for key in biased_dict:
        return key

The results of profiling them were:

  2226460 / s with first_1
  1905620 / s with first_2
  1994654 / s with first_3
  1777946 / s with first_4
  3681252 / s with first_5
  2829067 / s with first_6
  2600622 / s with first_7

All the approaches I tried are here:

def first_1():
    return next(iter(biased_dict))


def first_2():
    return list(biased_dict)[0]


def first_3():
    return next(iter(biased_dict.keys()))


def first_4():
    return list(biased_dict.keys())[0]


def first_5():
    for key in biased_dict:
        return key


def first_6():
    for key in biased_dict.keys():
        return key


def first_7():
    for key, v in biased_dict.items():
        return key

Automatically add all files in a folder to a target using CMake?

As of CMake 3.1+ the developers strongly discourage users from using file(GLOB or file(GLOB_RECURSE to collect lists of source files.

Note: We do not recommend using GLOB to collect a list of source files from your source tree. If no CMakeLists.txt file changes when a source is added or removed then the generated build system cannot know when to ask CMake to regenerate. The CONFIGURE_DEPENDS flag may not work reliably on all generators, or if a new generator is added in the future that cannot support it, projects using it will be stuck. Even if CONFIGURE_DEPENDS works reliably, there is still a cost to perform the check on every rebuild.

See the documentation here.

There are two goods answers ([1], [2]) here on SO detailing the reasons to manually list source files.


It is possible. E.g. with file(GLOB:

cmake_minimum_required(VERSION 2.8)

file(GLOB helloworld_SRC
     "*.h"
     "*.cpp"
)

add_executable(helloworld ${helloworld_SRC})

Note that this requires manual re-running of cmake if a source file is added or removed, since the generated build system does not know when to ask CMake to regenerate, and doing it at every build would increase the build time.

As of CMake 3.12, you can pass the CONFIGURE_DEPENDS flag to file(GLOB to automatically check and reset the file lists any time the build is invoked. You would write:

cmake_minimum_required(VERSION 3.12)

file(GLOB helloworld_SRC CONFIGURE_DEPENDS "*.h" "*.cpp")

This at least lets you avoid manually re-running CMake every time a file is added.

Create PDF from a list of images

If your images are in landscape mode, you can do like this.

from fpdf import FPDF
import os, sys, glob
from tqdm import tqdm

pdf = FPDF('L', 'mm', 'A4')
im_width = 1920
im_height = 1080

aspect_ratio = im_height/im_width
page_width = 297
# page_height = aspect_ratio * page_width
page_height = 200
left_margin = 0
right_margin = 0

# imagelist is the list with all image filenames
for image in tqdm(sorted(glob.glob('test_images/*.png'))):
pdf.add_page()
pdf.image(image, left_margin, right_margin, page_width, page_height)
pdf.output("mypdf.pdf", "F")
print('Conversion completed!')

Here page_width and page_height is the size of 'A4' paper where in landscape its width will 297mm and height will be 210mm; but here I have adjusted the height as per my image. OR you can use either maintaining the aspect ratio as I have commented above for proper scaling of both width and height of the image.

remove objects from array by object property

To delete an object by it's id in given array;

const hero = [{'id' : 1, 'name' : 'hero1'}, {'id': 2, 'name' : 'hero2'}];
//remove hero1
const updatedHero = hero.filter(item => item.id !== 1);

Convert HttpPostedFileBase to byte[]

As Darin says, you can read from the input stream - but I'd avoid relying on all the data being available in a single go. If you're using .NET 4 this is simple:

MemoryStream target = new MemoryStream();
model.File.InputStream.CopyTo(target);
byte[] data = target.ToArray();

It's easy enough to write the equivalent of CopyTo in .NET 3.5 if you want. The important part is that you read from HttpPostedFileBase.InputStream.

For efficient purposes you could check whether the stream returned is already a MemoryStream:

byte[] data;
using (Stream inputStream = model.File.InputStream)
{
    MemoryStream memoryStream = inputStream as MemoryStream;
    if (memoryStream == null)
    {
        memoryStream = new MemoryStream();
        inputStream.CopyTo(memoryStream);
    }
    data = memoryStream.ToArray();
}

How to delete $_POST variable upon pressing 'Refresh' button on browser with PHP?

This will remove the annoying confirm submission on refresh, the code is self-explanatory:

if (!isset($_SESSION)) {
session_start();
}

if ($_SERVER['REQUEST_METHOD'] == 'POST') {
$_SESSION['postdata'] = $_POST;
unset($_POST);
header("Location: ".$_SERVER[REQUEST_URI]);
exit;
}

if (@$_SESSION['postdata']){
$_POST=$_SESSION['postdata'];
unset($_SESSION['postdata']);
}

Java 8 - Difference between Optional.flatMap and Optional.map

What helped me was a look at the source code of the two functions.

Map - wraps the result in an Optional.

public<U> Optional<U> map(Function<? super T, ? extends U> mapper) {
    Objects.requireNonNull(mapper);
    if (!isPresent())
        return empty();
    else {
        return Optional.ofNullable(mapper.apply(value)); //<--- wraps in an optional
    }
}

flatMap - returns the 'raw' object

public<U> Optional<U> flatMap(Function<? super T, Optional<U>> mapper) {
    Objects.requireNonNull(mapper);
    if (!isPresent())
        return empty();
    else {
        return Objects.requireNonNull(mapper.apply(value)); //<---  returns 'raw' object
    }
}

Twitter Bootstrap 3, vertically center content

Option 1 is to use display:table-cell. You need to unfloat the Bootstrap col-* using float:none..

.center {
    display:table-cell;
    vertical-align:middle;
    float:none;
}

http://bootply.com/94394


Option 2 is display:flex to vertical align the row with flexbox:

.row.center {
   display: flex;
   align-items: center;
}

http://www.bootply.com/7rAuLpMCwr


Vertical centering is very different in Bootstrap 4. See this answer for Bootstrap 4 https://stackoverflow.com/a/41464397/171456

JQUERY: Uncaught Error: Syntax error, unrecognized expression

This can also happen in safari if you try a selector with a missing ], for example

$('select[name="something"')

but interestingly, this same jquery selector with a missing bracket will work in chrome.

Select top 1 result using JPA

Use a native SQL query by specifying a @NamedNativeQuery annotation on the entity class, or by using the EntityManager.createNativeQuery method. You will need to specify the type of the ResultSet using an appropriate class, or use a ResultSet mapping.

How to remove a class from elements in pure JavaScript?

Given worked for me.

document.querySelectorAll(".widget.hover").forEach(obj=>obj.classList.remove("hover"));

Hide particular div onload and then show div after click

$(document).ready(function() {
    $('#div2').hide(0);
    $('#preview').on('click', function() {
        $('#div1').hide(300, function() { // first hide div1
            // then show div2
            $('#div2').show(300);
        });     
    });
});

You missed # before div2

Working Sample

Render partial view with dynamic model in Razor view engine and ASP.NET MVC 3

I had the same problem & in my case this is what I did

@Html.Partial("~/Views/Cabinets/_List.cshtml", (List<Shop>)ViewBag.cabinets)

and in Partial view

@foreach (Shop cabinet in Model)
{
    //...
}

multiple ways of calling parent method in php

Unless I am misunderstanding the question, I would almost always use $this->get_species because the subclass (in this case dog) could overwrite that method since it does extend it. If the class dog doesn't redefine the method then both ways are functionally equivalent but if at some point in the future you decide you want the get_species method in dog should print "dog" then you would have to go back through all the code and change it.

When you use $this it is actually part of the object which you created and so will always be the most up-to-date as well (if the property being used has changed somehow in the lifetime of the object) whereas using the parent class is calling the static class method.

How do I use Wget to download all images into a single folder, from a URL?

wget -nd -r -l 2 -A jpg,jpeg,png,gif http://t.co
  • -nd: no directories (save all files to the current directory; -P directory changes the target directory)
  • -r -l 2: recursive level 2
  • -A: accepted extensions
wget -nd -H -p -A jpg,jpeg,png,gif -e robots=off example.tumblr.com/page/{1..2}
  • -H: span hosts (wget doesn't download files from different domains or subdomains by default)
  • -p: page requisites (includes resources like images on each page)
  • -e robots=off: execute command robotos=off as if it was part of .wgetrc file. This turns off the robot exclusion which means you ignore robots.txt and the robot meta tags (you should know the implications this comes with, take care).

Example: Get all .jpg files from an exemplary directory listing:

$ wget -nd -r -l 1 -A jpg http://example.com/listing/

Create Directory When Writing To File In Node.js

My advise is: try not to rely on dependencies when you can easily do it with few lines of codes

Here's what you're trying to achieve in 14 lines of code:

fs.isDir = function(dpath) {
    try {
        return fs.lstatSync(dpath).isDirectory();
    } catch(e) {
        return false;
    }
};
fs.mkdirp = function(dirname) {
    dirname = path.normalize(dirname).split(path.sep);
    dirname.forEach((sdir,index)=>{
        var pathInQuestion = dirname.slice(0,index+1).join(path.sep);
        if((!fs.isDir(pathInQuestion)) && pathInQuestion) fs.mkdirSync(pathInQuestion);
    });
};

What is the difference between up-casting and down-casting with respect to class variable

I know this question asked quite long time ago but for the new users of this question. Please read this article where contains complete description on upcasting, downcasting and use of instanceof operator

  • There's no need to upcast manually, it happens on its own:

    Mammal m = (Mammal)new Cat(); equals to Mammal m = new Cat();

  • But downcasting must always be done manually:

    Cat c1 = new Cat();      
    Animal a = c1;      //automatic upcasting to Animal
    Cat c2 = (Cat) a;    //manual downcasting back to a Cat
    

Why is that so, that upcasting is automatical, but downcasting must be manual? Well, you see, upcasting can never fail. But if you have a group of different Animals and want to downcast them all to a Cat, then there's a chance, that some of these Animals are actually Dogs, and process fails, by throwing ClassCastException. This is where is should introduce an useful feature called "instanceof", which tests if an object is instance of some Class.

 Cat c1 = new Cat();         
    Animal a = c1;       //upcasting to Animal
    if(a instanceof Cat){ // testing if the Animal is a Cat
        System.out.println("It's a Cat! Now i can safely downcast it to a Cat, without a fear of failure.");        
        Cat c2 = (Cat)a;
    }

For more information please read this article

How to generate xsd from wsdl

Once I found an xsd link on the top of the wsdl. Like this wsdl example from the web, you can see a link xsd1. The server has to be running to see it.

<?xml version="1.0"?>
<definitions name="StockQuote"
             targetNamespace="http://example.com/stockquote.wsdl"
             xmlns:tns="http://example.com/stockquote.wsdl"
             xmlns:xsd1="http://example.com/stockquote.xsd"
             xmlns:soap="http://schemas.xmlsoap.org/wsdl/soap/"
             xmlns="http://schemas.xmlsoap.org/wsdl/">

Get name of currently executing test in JUnit 4

JUnit 4 does not have any out-of-the-box mechanism for a test case to get it’s own name (including during setup and teardown).

Git: How to remove file from index without deleting files from any repository

The above solutions work fine for most cases. However, if you also need to remove all traces of that file (ie sensitive data such as passwords), you will also want to remove it from your entire commit history, as the file could still be retrieved from there.

Here is a solution that removes all traces of the file from your entire commit history, as though it never existed, yet keeps the file in place on your system.

https://help.github.com/articles/remove-sensitive-data/

You can actually skip to step 3 if you are in your local git repository, and don't need to perform a dry run. In my case, I only needed steps 3 and 6, as I had already created my .gitignore file, and was in the repository I wanted to work on.

To see your changes, you may need to go to the GitHub root of your repository and refresh the page. Then navigate through the links to get to an old commit that once had the file, to see that it has now been removed. For me, simply refreshing the old commit page did not show the change.

It looked intimidating at first, but really, was easy and worked like a charm ! :-)

How to deal with SettingWithCopyWarning in Pandas

This should work:

quote_df.loc[:,'TVol'] = quote_df['TVol']/TVOL_SCALE

The name 'model' does not exist in current context in MVC3

Had similar problems using VS2012 and VS2013.
Adding the following line to <appSettings> in the main web.config worked:

<add key="webpages:Version" value="3.0.0.0" />

If the line was already there but said 2.0.0.0, changing it to 3.0.0.0 worked.

Set select option 'selected', by value

Just put in this code:

$("#Controls_ID").val(value);

Make elasticsearch only return certain fields?

I found the docs for the get api to be helpful - especially the two sections, Source filtering and Fields: https://www.elastic.co/guide/en/elasticsearch/reference/7.3/docs-get.html#get-source-filtering

They state about source filtering:

If you only need one or two fields from the complete _source, you can use the _source_include & _source_exclude parameters to include or filter out that parts you need. This can be especially helpful with large documents where partial retrieval can save on network overhead

Which fitted my use case perfectly. I ended up simply filtering the source like so (using the shorthand):

{
    "_source": ["field_x", ..., "field_y"],
    "query": {      
        ...
    }
}

FYI, they state in the docs about the fields parameter:

The get operation allows specifying a set of stored fields that will be returned by passing the fields parameter.

It seems to cater for fields that have been specifically stored, where it places each field in an array. If the specified fields haven't been stored it will fetch each one from the _source, which could result in 'slower' retrievals. I also had trouble trying to get it to return fields of type object.

So in summary, you have two options, either though source filtering or [stored] fields.

How to make sure that a certain Port is not occupied by any other process

It's netstat -ano|findstr port no

Result would show process id in last column

Android Log.v(), Log.d(), Log.i(), Log.w(), Log.e() - When to use each one?

The source code provides some basic guidance:

The order in terms of verbosity, from least to most is ERROR, WARN, INFO, DEBUG, VERBOSE. Verbose should never be compiled into an application except during development. Debug logs are compiled in but stripped at runtime. Error, warning and info logs are always kept.

For more detail, Kurtis' answer is dead on. I would just add: Don't log any personally identifiable or private information at INFO or above (WARN/ERROR). Otherwise, bug reports or anything else that includes logging may be polluted.

How to read values from properties file?

 [project structure]: http://i.stack.imgur.com/RAGX3.jpg
-------------------------------
    package beans;

        import java.util.Properties;
        import java.util.Set;

        public class PropertiesBeans {

            private Properties properties;

            public void setProperties(Properties properties) {
                this.properties = properties;
            }

            public void getProperty(){
                Set keys = properties.keySet();
                for (Object key : keys) {
                    System.out.println(key+" : "+properties.getProperty(key.toString()));
                }
            }

        }
    ----------------------------

        package beans;

        import org.springframework.context.ApplicationContext;
        import org.springframework.context.support.ClassPathXmlApplicationContext;

        public class Test {

            public static void main(String[] args) {
                // TODO Auto-generated method stub
                ApplicationContext ap = new ClassPathXmlApplicationContext("resource/spring.xml");
                PropertiesBeans p = (PropertiesBeans)ap.getBean("p");
                p.getProperty();
            }

        }
    ----------------------------

 - driver.properties

    Driver = com.mysql.jdbc.Driver
    url = jdbc:mysql://localhost:3306/test
    username = root
    password = root
    ----------------------------



     <beans xmlns="http://www.springframework.org/schema/beans"
               xmlns:xsi="http://www.w3.org/2001/XMLSchema-instance"
               xmlns:util="http://www.springframework.org/schema/util"
               xsi:schemaLocation="
        http://www.springframework.org/schema/beans http://www.springframework.org/schema/beans/spring-beans-3.0.xsd
        http://www.springframework.org/schema/util http://www.springframework.org/schema/util/spring-util-3.0.xsd">

            <bean id="p" class="beans.PropertiesBeans">
                <property name="properties">
                    <util:properties location="classpath:resource/driver.properties"/>
                </property>
            </bean>

        </beans>

Wait 5 seconds before executing next line

Best way to create a function like this for wait in milli seconds, this function will wait for milliseconds provided in the argument:

_x000D_
_x000D_
function waitSeconds(iMilliSeconds) {_x000D_
    var counter= 0_x000D_
        , start = new Date().getTime()_x000D_
        , end = 0;_x000D_
    while (counter < iMilliSeconds) {_x000D_
        end = new Date().getTime();_x000D_
        counter = end - start;_x000D_
    }_x000D_
}
_x000D_
_x000D_
_x000D_

CSS text-overflow: ellipsis; not working?

In bootstrap 4, you can add a .text-truncate class to truncate the text with an ellipsis.

_x000D_
_x000D_
<script src="https://ajax.googleapis.com/ajax/libs/jquery/2.1.1/jquery.min.js"></script>_x000D_
<link href="https://stackpath.bootstrapcdn.com/bootstrap/4.1.3/css/bootstrap.min.css" rel="stylesheet" />_x000D_
<script src="https://stackpath.bootstrapcdn.com/bootstrap/4.1.3/js/bootstrap.min.js"></script>_x000D_
_x000D_
<!-- Inline level -->_x000D_
<span class="d-inline-block text-truncate" style="max-width: 250px;">_x000D_
  The quick brown fox jumps over the lazy dog._x000D_
</span>
_x000D_
_x000D_
_x000D_

How can I convert tabs to spaces in every file of a directory?

Try the command line tool expand.

expand -i -t 4 input | sponge output

where

  • -i is used to expand only leading tabs on each line;
  • -t 4 means that each tab will be converted to 4 whitespace chars (8 by default).
  • sponge is from the moreutils package, and avoids clearing the input file.

Finally, you can use gexpand on OSX, after installing coreutils with Homebrew (brew install coreutils).

Java JRE 64-bit download for Windows?

Might this be the download you are looking for?

  1. Go to the Java SE Downloads Page.
  2. Scroll down a tad look for the main table with the header of "Java Platform, Standard Edition"
  3. Click the JRE Download Button (JRE is the runtime component. JDK is the developer's kit).
  4. Select the appropriate download (all platforms and 32/64 bit downloads are listed)

MySQL: ERROR 1227 (42000): Access denied - Cannot CREATE USER

First thing to do is run this:

SHOW GRANTS;

You will quickly see you were assigned the anonymous user to authenticate into mysql.

Instead of logging into mysql with

mysql

login like this:

mysql -uroot

By default, root@localhost has all rights and no password.

If you cannot login as root without a password, do the following:

Step 01) Add the two options in the mysqld section of my.ini:

[mysqld]
skip-grant-tables
skip-networking

Step 02) Restart mysql

net stop mysql
<wait 10 seconds>
net start mysql

Step 03) Connect to mysql

mysql

Step 04) Create a password from root@localhost

UPDATE mysql.user SET password=password('whateverpasswordyoulike')
WHERE user='root' AND host='localhost';
exit

Step 05) Restart mysql

net stop mysql
<wait 10 seconds>
net start mysql

Step 06) Login as root with password

mysql -u root -p

You should be good from there.

CAVEAT: Please remove anonymous users !!!

Invisible characters - ASCII

I just went through the character map to get these. They are all in Calibri.

Number    Name                   HTML Code    Appearance
------    --------------------   ---------    ----------
U+2000    En Quad                &#8192;      " "
U+2001    Em Quad                &#8193;      " "
U+2002    En Space               &#8194;      " "
U+2003    Em Space               &#8195;      " "
U+2004    Three-Per-Em Space     &#8196;      " "
U+2005    Four-Per-Em Space      &#8197;      " "
U+2006    Six-Per-Em Space       &#8198;      " "
U+2007    Figure Space           &#8199;      " "
U+2008    Punctuation Space      &#8200;      " "
U+2009    Thin Space             &#8201;      " "
U+200A    Hair Space             &#8202;      " "
U+200B    Zero-Width Space       &#8203;      "​"
U+200C    Zero Width Non-Joiner  &#8204;      "‌"
U+200D    Zero Width Joiner      &#8205;      "‍"
U+200E    Left-To-Right Mark     &#8206;      "‎"
U+200F    Right-To-Left Mark     &#8207;      "‏"
U+202F    Narrow No-Break Space  &#8239;      " "

What is the difference between HTTP and REST?

From You don't know the difference between HTTP and REST

So REST architecture and HTTP 1.1 protocol are independent from each other, but the HTTP 1.1 protocol was built to be the ideal protocol to follow the principles and constraints of REST. One way to look at the relationship between HTTP and REST is, that REST is the design, and HTTP 1.1 is an implementation of that design.

How to get the size of a varchar[n] field in one SQL statement?

On SQL Server specifically:

SELECT DATALENGTH(Remarks) AS FIELDSIZE FROM mytable

Documentation

How to clear all input fields in a specific div with jQuery?

Change this:

 <div class=fetch_results>

To this:

 <div id="fetch_results">

Then the following should work:

$("#fetch_results input").each(function() {
      this.value = "";
  })?

http://jsfiddle.net/NVqeR/

How to set 24-hours format for date on java?

Use HH instead of hh in formatter string

Is using 'var' to declare variables optional?

Consider this question asked at StackOverflow today:

Simple Javascript question

A good test and a practical example is what happens in the above scenario...
The developer used the name of the JavaScript function in one of his variables.

What's the problem with the code?
The code only works the first time the user clicks the button.

What's the solution?
Add the var keyword before the variable name.

How to disable 'X-Frame-Options' response header in Spring Security?

If using XML configuration you can use

<beans xmlns="http://www.springframework.org/schema/beans" 
       xmlns:security="http://www.springframework.org/schema/security"> 
<security:http>
    <security:headers>
         <security:frame-options disabled="true"></security:frame-options>
    </security:headers>
</security:http>
</beans>

Bootstrap 3: How do you align column content to bottom of row

I don't know why but for me the solution proposed by Marius Stanescu is breaking the specificity of col (a col-md-3 followed by a col-md-4 will take all of the twelve row)

I found another working solution :

.bottom-column 
{
   display: inline-block;
   vertical-align: middle;
   float: none;
}

Element-wise addition of 2 lists?

Perhaps "the most pythonic way" should include handling the case where list1 and list2 are not the same size. Applying some of these methods will quietly give you an answer. The numpy approach will let you know, most likely with a ValueError.

Example:

import numpy as np
>>> list1 = [ 1, 2 ]
>>> list2 = [ 1, 2, 3]
>>> list3 = [ 1 ]
>>> [a + b for a, b in zip(list1, list2)]
[2, 4]
>>> [a + b for a, b in zip(list1, list3)]
[2]
>>> a = np.array (list1)
>>> b = np.array (list2)
>>> a+b
Traceback (most recent call last):
  File "<stdin>", line 1, in <module>
ValueError: operands could not be broadcast together with shapes (2) (3)

Which result might you want if this were in a function in your problem?

Docker compose port mapping

If you want to access redis from the host (127.0.0.1), you have to use the ports command.

redis:
  build:
    context: .
    dockerfile: Dockerfile-redis
    ports:
    - "6379:6379"

How to drop a PostgreSQL database if there are active connections to it?

Nothing worked for me except, I loggined using pgAdmin4 and on the Dashboard I disconnected all connections except pgAdmin4 and then was able to rename by right lick on the database and properties and typed new name.

How can I change the app display name build with Flutter?

You can change it in iOS without opening Xcode by editing file *project/ios/Runner/info.plist. Set <key>CFBundleDisplayName</key> to the string that you want as your name.

For Android, change the app name from the Android folder, in the AndroidManifest.xml file, android/app/src/main. Let the android label refer to the name you prefer, for example,

<manifest xmlns:android="http://schemas.android.com/apk/res/android"
    <application
        android:label="test"
        // The rest of the code
    </application>
</manifest>

How to install both Python 2.x and Python 3.x in Windows

I found that the formal way to do this is as follows:

Just install two (or more, using their installers) versions of Python on Windows 7 (for me work with 3.3 and 2.7).

Follow the instuctions below, changing the parameters for your needs.

Create the following environment variable (to default on double click):

Name:  PY_PYTHON
Value: 3

To launch a script in a particular interpreter, add the following shebang (beginning of script):

#! python2

To execute a script using a specific interpreter, use the following prompt command:

> py -2 MyScript.py

To launch a specific interpreter:

> py -2

To launch the default interpreter (defined by the PY_PYTHON variable):

> py

Resources

Documentation: Using Python on Windows

PEP 397 - Python launcher for Windows

How to create empty folder in java?

You can create folder using the following Java code:

File dir = new File("nameoffolder");
dir.mkdir();

By executing above you will have folder 'nameoffolder' in current folder.

How do I convert Int/Decimal to float in C#?

The same as an int:

float f = 6;

Also here's how to programmatically convert from an int to a float, and a single in C# is the same as a float:

int i = 8;
float f = Convert.ToSingle(i);

Or you can just cast an int to a float:

float f = (float)i;

How to convert a hex string to hex number

Try this:

hex_str = "0xAD4"
hex_int = int(hex_str, 16)
new_int = hex_int + 0x200
print hex(new_int)

If you don't like the 0x in the beginning, replace the last line with

print hex(new_int)[2:]

while ($row = mysql_fetch_array($result)) - how many loops are being performed?

It depends how many rows are returned in $results, and how many columns there are in $row?

How do I find the caller of a method using stacktrace or reflection?

I've done this before. You can just create a new exception and grab the stack trace on it without throwing it, then examine the stack trace. As the other answer says though, it's extremely costly--don't do it in a tight loop.

I've done it before for a logging utility on an app where performance didn't matter much (Performance rarely matters much at all, actually--as long as you display the result to an action such as a button click quickly).

It was before you could get the stack trace, exceptions just had .printStackTrace() so I had to redirect System.out to a stream of my own creation, then (new Exception()).printStackTrace(); Redirect System.out back and parse the stream. Fun stuff.

error: expected unqualified-id before ‘.’ token //(struct)

The struct's name is ReducedForm; you need to make an object (instance of the struct or class) and use that. Do this:

ReducedForm MyReducedForm;
MyReducedForm.iSimplifiedNumerator = iNumerator/iGreatCommDivisor;
MyReducedForm.iSimplifiedDenominator = iDenominator/iGreatCommDivisor;

Objects inside objects in javascript

You may have as many levels of Object hierarchy as you want, as long you declare an Object as being a property of another parent Object. Pay attention to the commas on each level, that's the tricky part. Don't use commas after the last element on each level:

{el1, el2, {el31, el32, el33}, {el41, el42}}

_x000D_
_x000D_
var MainObj = {_x000D_
_x000D_
  prop1: "prop1MainObj",_x000D_
  _x000D_
  Obj1: {_x000D_
    prop1: "prop1Obj1",_x000D_
    prop2: "prop2Obj1",    _x000D_
    Obj2: {_x000D_
      prop1: "hey you",_x000D_
      prop2: "prop2Obj2"_x000D_
    }_x000D_
  },_x000D_
    _x000D_
  Obj3: {_x000D_
    prop1: "prop1Obj3",_x000D_
    prop2: "prop2Obj3"_x000D_
  },_x000D_
  _x000D_
  Obj4: {_x000D_
    prop1: true,_x000D_
    prop2: 3_x000D_
  }  _x000D_
};_x000D_
_x000D_
console.log(MainObj.Obj1.Obj2.prop1);
_x000D_
_x000D_
_x000D_

When to use static keyword before global variables?

global static variables are initialized at compile-time unlike automatic

Setting public class variables

You're "setting" the value of that variable/attribute. Not overriding or overloading it. Your code is very, very common and normal.

All of these terms ("set", "override", "overload") have specific meanings. Override and Overload are about polymorphism (subclassing).

From http://en.wikipedia.org/wiki/Object-oriented_programming :

Polymorphism allows the programmer to treat derived class members just like their parent class' members. More precisely, Polymorphism in object-oriented programming is the ability of objects belonging to different data types to respond to method calls of methods of the same name, each one according to an appropriate type-specific behavior. One method, or an operator such as +, -, or *, can be abstractly applied in many different situations. If a Dog is commanded to speak(), this may elicit a bark(). However, if a Pig is commanded to speak(), this may elicit an oink(). They both inherit speak() from Animal, but their derived class methods override the methods of the parent class; this is Overriding Polymorphism. Overloading Polymorphism is the use of one method signature, or one operator such as "+", to perform several different functions depending on the implementation. The "+" operator, for example, may be used to perform integer addition, float addition, list concatenation, or string concatenation. Any two subclasses of Number, such as Integer and Double, are expected to add together properly in an OOP language. The language must therefore overload the addition operator, "+", to work this way. This helps improve code readability. How this is implemented varies from language to language, but most OOP languages support at least some level of overloading polymorphism.

How to access JSON decoded array in PHP

$data = json_decode($json, true);
echo $data[0]["c_name"]; // "John"


$data = json_decode($json);
echo $data[0]->c_name;      // "John"

Business logic in MVC

Why don't you introduce a service layer. then your controller will be lean and more readable, then your all controller functions will be pure actions. you can decompose business logic as you much as you need within service layer . code reusability is hight . no impact on models and repositories.

What does int argc, char *argv[] mean?

int main();

This is a simple declaration. It cannot take any command line arguments.

int main(int argc, char* argv[]);

This declaration is used when your program must take command-line arguments. When run like such:

myprogram arg1 arg2 arg3

argc, or Argument Count, will be set to 4 (four arguments), and argv, or Argument Vectors, will be populated with string pointers to "myprogram", "arg1", "arg2", and "arg3". The program invocation (myprogram) is included in the arguments!

Alternatively, you could use:

int main(int argc, char** argv);

This is also valid.

There is another parameter you can add:

int main (int argc, char *argv[], char *envp[])

The envp parameter also contains environment variables. Each entry follows this format:

VARIABLENAME=VariableValue

like this:

SHELL=/bin/bash    

The environment variables list is null-terminated.

IMPORTANT: DO NOT use any argv or envp values directly in calls to system()! This is a huge security hole as malicious users could set environment variables to command-line commands and (potentially) cause massive damage. In general, just don't use system(). There is almost always a better solution implemented through C libraries.

Two's Complement in Python

This works for 3 bytes. Live code is here

def twos_compliment(byte_arr):
   a = byte_arr[0]; b = byte_arr[1]; c = byte_arr[2]
   out = ((a<<16)&0xff0000) | ((b<<8)&0xff00) | (c&0xff)
   neg = (a & (1<<7) != 0)  # first bit of a is the "signed bit." if it's a 1, then the value is negative
   if neg: out -= (1 << 24)
   print(hex(a), hex(b), hex(c), neg, out)
   return out


twos_compliment([0x00, 0x00, 0x01])
>>> 1

twos_compliment([0xff,0xff,0xff])
>>> -1

twos_compliment([0b00010010, 0b11010110, 0b10000111])
>>> 1234567

twos_compliment([0b11101101, 0b00101001, 0b01111001])
>>> -1234567

twos_compliment([0b01110100, 0b11001011, 0b10110001])
>>> 7654321

twos_compliment([0b10001011, 0b00110100, 0b01001111])
>>> -7654321

Difference in boto3 between resource, client, and session?

I'll try and explain it as simple as possible. So there is no guarantee of the accuracy of the actual terms.

Session is where to initiate the connectivity to AWS services. E.g. following is default session that uses the default credential profile(e.g. ~/.aws/credentials, or assume your EC2 using IAM instance profile )

sqs = boto3.client('sqs')
s3 = boto3.resource('s3')

Because default session is limit to the profile or instance profile used, sometimes you need to use the custom session to override the default session configuration (e.g. region_name, endpoint_url, etc. ) e.g.

# custom resource session must use boto3.Session to do the override
my_west_session = boto3.Session(region_name = 'us-west-2')
my_east_session = boto3.Session(region_name = 'us-east-1')
backup_s3 = my_west_session.resource('s3')
video_s3 = my_east_session.resource('s3')

# you have two choices of create custom client session. 
backup_s3c = my_west_session.client('s3')
video_s3c = boto3.client("s3", region_name = 'us-east-1')

Resource : This is the high-level service class recommended to be used. This allows you to tied particular AWS resources and passes it along, so you just use this abstraction than worry which target services are pointed to. As you notice from the session part, if you have a custom session, you just pass this abstract object than worrying about all custom regions,etc to pass along. Following is a complicated example E.g.

import boto3 
my_west_session = boto3.Session(region_name = 'us-west-2')
my_east_session = boto3.Session(region_name = 'us-east-1')
backup_s3 = my_west_session.resource("s3")
video_s3 = my_east_session.resource("s3")
backup_bucket = backup_s3.Bucket('backupbucket') 
video_bucket = video_s3.Bucket('videobucket')

# just pass the instantiated bucket object
def list_bucket_contents(bucket):
   for object in bucket.objects.all():
      print(object.key)

list_bucket_contents(backup_bucket)
list_bucket_contents(video_bucket)

Client is a low level class object. For each client call, you need to explicitly specify the targeting resources, the designated service target name must be pass long. You will lose the abstraction ability.

For example, if you only deal with the default session, this looks similar to boto3.resource.

import boto3 
s3 = boto3.client('s3')

def list_bucket_contents(bucket_name):
   for object in s3.list_objects_v2(Bucket=bucket_name) :
      print(object.key)

list_bucket_contents('Mybucket') 

However, if you want to list objects from a bucket in different regions, you need to specify the explicit bucket parameter required for the client.

import boto3 
backup_s3 = my_west_session.client('s3',region_name = 'us-west-2')
video_s3 = my_east_session.client('s3',region_name = 'us-east-1')

# you must pass boto3.Session.client and the bucket name 
def list_bucket_contents(s3session, bucket_name):
   response = s3session.list_objects_v2(Bucket=bucket_name)
   if 'Contents' in response:
     for obj in response['Contents']:
        print(obj['key'])

list_bucket_contents(backup_s3, 'backupbucket')
list_bucket_contents(video_s3 , 'videobucket') 

Change the value in app.config file dynamically

 XmlReaderSettings _configsettings = new XmlReaderSettings();
 _configsettings.IgnoreComments = true;

 XmlReader _configreader = XmlReader.Create(ConfigFilePath, _configsettings);
 XmlDocument doc_config = new XmlDocument();
 doc_config.Load(_configreader);
 _configreader.Close();

 foreach (XmlNode RootName in doc_config.DocumentElement.ChildNodes)
 {
     if (RootName.LocalName == "appSettings")
     {
         if (RootName.HasChildNodes)
         {
             foreach (XmlNode _child in RootName.ChildNodes)
             {
                 if (_child.Attributes["key"].Value == "HostName")
                 {
                     if (_child.Attributes["value"].Value == "false")
                         _child.Attributes["value"].Value = "true";
                 }
             }
         }
     }
 }
 doc_config.Save(ConfigFilePath);

width:auto for <input> fields

An <input>'s width is generated from its size attribute. The default size is what's driving the auto width.

You could try width:100% as illustrated in my example below.

Doesn't fill width:

<form action='' method='post' style='width:200px;background:khaki'>
  <input style='width:auto' />
</form>

Fills width:

<form action='' method='post' style='width:200px;background:khaki'>
  <input style='width:100%' />
</form>

Smaller size, smaller width:

<form action='' method='post' style='width:200px;background:khaki'>
  <input size='5' />
</form>

UPDATE

Here's the best I could do after a few minutes. It's 1px off in FF, Chrome, and Safari, and perfect in IE. (The problem is #^&* IE applies borders differently than everyone else so it's not consistent.)

<div style='padding:30px;width:200px;background:red'>
  <form action='' method='post' style='width:200px;background:blue;padding:3px'>
    <input size='' style='width:100%;margin:-3px;border:2px inset #eee' />
    <br /><br />
    <input size='' style='width:100%' />
  </form>
</div>

Android on-screen keyboard auto popping up

InputMethodManager imm = (InputMethodManager)GetSystemService(Context.InputMethodService);
        imm.ShowSoftInput(_enterPin.FindFocus(), 0);

*This is for Android.xamarin and FindFocus()-it searches for the view in hierarchy rooted at this view that currently has focus,as i have _enterPin.RequestFocus() before the above code thus it shows keyboard for _enterPin EditText *

Jquery assiging class to th in a table

You had thead in your selector, but there is no thead in your table. Also you had your selectors backwards. As you mentioned above, you wanted to be adding the tr class to the th, not vice-versa (although your comment seems to contradict what you wrote up above).

$('tr th').each(function(index){     if($('tr td').eq(index).attr('class') != ''){         // get the class of the td         var tdClass = $('tr td').eq(index).attr('class');         // add it to this th         $(this).addClass(tdClass );     } }); 

Fiddle

Random character generator with a range of (A..Z, 0..9) and punctuation

Here is code for secure, easy, but a little bit more expensive session identifiers.

import java.security.SecureRandom;
import java.math.BigInteger;

public final class SessionIdentifierGenerator
{

  private SecureRandom random = new SecureRandom();

  public String nextSessionId()
  {
    return new BigInteger(130, random).toString(32);
  }

}

Are the shift operators (<<, >>) arithmetic or logical in C?

will typically use logical shifts on unsigned variables and for left-shifts on signed variables. The arithmetic right shift is the truly important one because it will sign extend the variable.

will will use this when applicable, as other compilers are likely to do.

Performing user authentication in Java EE / JSF using j_security_check

I suppose you want form based authentication using deployment descriptors and j_security_check.

You can also do this in JSF by just using the same predefinied field names j_username and j_password as demonstrated in the tutorial.

E.g.

<form action="j_security_check" method="post">
    <h:outputLabel for="j_username" value="Username" />
    <h:inputText id="j_username" />
    <br />
    <h:outputLabel for="j_password" value="Password" />
    <h:inputSecret id="j_password" />
    <br />
    <h:commandButton value="Login" />
</form>

You could do lazy loading in the User getter to check if the User is already logged in and if not, then check if the Principal is present in the request and if so, then get the User associated with j_username.

package com.stackoverflow.q2206911;

import java.io.IOException;
import java.security.Principal;

import javax.faces.bean.ManagedBean;
import javax.faces.bean.SessionScoped;
import javax.faces.context.FacesContext;

@ManagedBean
@SessionScoped
public class Auth {

    private User user; // The JPA entity.

    @EJB
    private UserService userService;

    public User getUser() {
        if (user == null) {
            Principal principal = FacesContext.getCurrentInstance().getExternalContext().getUserPrincipal();
            if (principal != null) {
                user = userService.find(principal.getName()); // Find User by j_username.
            }
        }
        return user;
    }

}

The User is obviously accessible in JSF EL by #{auth.user}.

To logout do a HttpServletRequest#logout() (and set User to null!). You can get a handle of the HttpServletRequest in JSF by ExternalContext#getRequest(). You can also just invalidate the session altogether.

public String logout() {
    FacesContext.getCurrentInstance().getExternalContext().invalidateSession();
    return "login?faces-redirect=true";
}

For the remnant (defining users, roles and constraints in deployment descriptor and realm), just follow the Java EE 6 tutorial and the servletcontainer documentation the usual way.


Update: you can also use the new Servlet 3.0 HttpServletRequest#login() to do a programmatic login instead of using j_security_check which may not per-se be reachable by a dispatcher in some servletcontainers. In this case you can use a fullworthy JSF form and a bean with username and password properties and a login method which look like this:

<h:form>
    <h:outputLabel for="username" value="Username" />
    <h:inputText id="username" value="#{auth.username}" required="true" />
    <h:message for="username" />
    <br />
    <h:outputLabel for="password" value="Password" />
    <h:inputSecret id="password" value="#{auth.password}" required="true" />
    <h:message for="password" />
    <br />
    <h:commandButton value="Login" action="#{auth.login}" />
    <h:messages globalOnly="true" />
</h:form>

And this view scoped managed bean which also remembers the initially requested page:

@ManagedBean
@ViewScoped
public class Auth {

    private String username;
    private String password;
    private String originalURL;

    @PostConstruct
    public void init() {
        ExternalContext externalContext = FacesContext.getCurrentInstance().getExternalContext();
        originalURL = (String) externalContext.getRequestMap().get(RequestDispatcher.FORWARD_REQUEST_URI);

        if (originalURL == null) {
            originalURL = externalContext.getRequestContextPath() + "/home.xhtml";
        } else {
            String originalQuery = (String) externalContext.getRequestMap().get(RequestDispatcher.FORWARD_QUERY_STRING);

            if (originalQuery != null) {
                originalURL += "?" + originalQuery;
            }
        }
    }

    @EJB
    private UserService userService;

    public void login() throws IOException {
        FacesContext context = FacesContext.getCurrentInstance();
        ExternalContext externalContext = context.getExternalContext();
        HttpServletRequest request = (HttpServletRequest) externalContext.getRequest();

        try {
            request.login(username, password);
            User user = userService.find(username, password);
            externalContext.getSessionMap().put("user", user);
            externalContext.redirect(originalURL);
        } catch (ServletException e) {
            // Handle unknown username/password in request.login().
            context.addMessage(null, new FacesMessage("Unknown login"));
        }
    }

    public void logout() throws IOException {
        ExternalContext externalContext = FacesContext.getCurrentInstance().getExternalContext();
        externalContext.invalidateSession();
        externalContext.redirect(externalContext.getRequestContextPath() + "/login.xhtml");
    }

    // Getters/setters for username and password.
}

This way the User is accessible in JSF EL by #{user}.

Is it better to use std::memcpy() or std::copy() in terms to performance?

If you really need maximum copying performance (which you might not), use neither of them.

There's a lot that can be done to optimize memory copying - even more if you're willing to use multiple threads/cores for it. See, for example:

What's missing/sub-optimal in this memcpy implementation?

both the question and some of the answers have suggested implementations or links to implementations.

java.sql.SQLException: Missing IN or OUT parameter at index:: 1

You must use the column names and then set the values to insert (both ? marks):

//insert 1st row            
String inserting = "INSERT INTO employee(emp_name ,emp_address) values(?,?)";
System.out.println("insert " + inserting);//
PreparedStatement ps = con.prepareStatement(inserting); 
ps.setString(1, "hans");
ps.setString(2, "germany");
ps.executeUpdate();

SQL Server table creation date query

For SQL Server 2000:

SELECT   su.name,so.name,so.crdate,* 
FROM     sysobjects so JOIN sysusers su
ON       so.uid = su.uid
WHERE    xtype='U'
ORDER BY so.name

java.net.URL read stream to byte[]

I am very surprised that nobody here has mentioned the problem of connection and read timeout. It could happen (especially on Android and/or with some crappy network connectivity) that the request will hang and wait forever.

The following code (which also uses Apache IO Commons) takes this into account, and waits max. 5 seconds until it fails:

public static byte[] downloadFile(URL url)
{
    try {
        URLConnection conn = url.openConnection();
        conn.setConnectTimeout(5000);
        conn.setReadTimeout(5000);
        conn.connect(); 

        ByteArrayOutputStream baos = new ByteArrayOutputStream();
        IOUtils.copy(conn.getInputStream(), baos);

        return baos.toByteArray();
    }
    catch (IOException e)
    {
        // Log error and return null, some default or throw a runtime exception
    }
}

Add a duration to a moment (moment.js)

I think you missed a key point in the documentation for .add()

Mutates the original moment by adding time.

You appear to be treating it as a function that returns the immutable result. Easy mistake to make. :)

If you use the return value, it is the same actual object as the one you started with. It's just returned as a convenience for method chaining.

You can work around this behavior by cloning the moment, as described here.

Also, you cannot just use == to test. You could format each moment to the same output and compare those, or you could just use the .isSame() method.

Your code is now:

var timestring1 = "2013-05-09T00:00:00Z";
var timestring2 = "2013-05-09T02:00:00Z";
var startdate = moment(timestring1);
var expected_enddate = moment(timestring2);
var returned_endate = moment(startdate).add(2, 'hours');  // see the cloning?
returned_endate.isSame(expected_enddate)  // true

Passing an Object from an Activity to a Fragment

Passing arguments by bundle is restricted to some data types. But you can transfer any data to your fragment this way:

In your fragment create a public method like this

public void passData(Context context, List<LexItem> list, int pos) {
    mContext = context;
    mLexItemList = list;
    mIndex = pos;
}

and in your activity call passData() with all your needed data types after instantiating the fragment

        WebViewFragment myFragment = new WebViewFragment();
        myFragment.passData(getApplicationContext(), mLexItemList, index);
        FragmentManager fm = getSupportFragmentManager();
        FragmentTransaction ft = fm.beginTransaction();
        ft.add(R.id.my_fragment_container, myFragment);
        ft.addToBackStack(null);
        ft.commit();

Remark: My fragment extends "android.support.v4.app.Fragment", therefore I have to use "getSupportFragmentManager()". Of course, this principle will work also with a fragment class extending "Fragment", but then you have to use "getFragmentManager()".

Why is char[] preferred over String for passwords?

It is debatable as to whether you should use String or use Char[] for this purpose because both have their advantages and disadvantages. It depends on what the user needs.

Since Strings in Java are immutable, whenever some tries to manipulate your string it creates a new Object and the existing String remains unaffected. This could be seen as an advantage for storing a password as a String, but the object remains in memory even after use. So if anyone somehow got the memory location of the object, that person can easily trace your password stored at that location.

Char[] is mutable, but it has the advantage that after its usage the programmer can explicitly clean the array or override values. So when it's done being used it is cleaned and no one could ever know about the information you had stored.

Based on the above circumstances, one can get an idea whether to go with String or to go with Char[] for their requirements.

How to start Apache and MySQL automatically when Windows 8 comes up

One of the latest XAMPP releases (XAMPP for Windows v5.6.11 (PHP 5.6.11) for sure, probably some earlier versions too) does not have the Control Panel with the "Svc" checkbox that allows to install Apache and MySQL as a service.

Go to your XAMPP/Apache directory instead (typically C:/xampp/apache) and run apache_installservice.bat as an administrator. There is also apache_uninstallservice.bat for uninstall.

To run MySQL as a service. Do it the same way - the location is xampp/mysql and batch files are: mysql_installservice.bat for service installation and mysql_uninstallservice.bat for removing the MySQL service.

You can check if they were installed or not by going to services manager window (press Windows + R and type: services.msc) and check if you have Apache service (I had Apache2.4) running and set to startup automatically. The MySQL service name is just: mysql.

mongodb how to get max value from collections

db.collection.findOne().sort({age:-1}) //get Max without need for limit(1)

Performance differences between ArrayList and LinkedList

I want to add an additional piece of information her about the difference in performance.

We already know that due to the fact that ArrayList implementation is backed by an Object[] it's supports random access and dynamic resizing and LinkedList implementation uses references to head and tail to navigate it. It has no random access capabilities, but it supports dynamic resizing as well.

The first thing is that with an ArrayList, you can immediately access the index, whereas with a LinkedList, you have iterate down the object chain.

Secondly, inserting into ArrayList is generally slower because it has to grow once you hit its boundaries. It will have to create a new bigger array, and copy data from the original one.

But the interesting thing is that when you create an ArrayList that is already huge enough to fit all your inserts it will obviously not involve any array copying operations. Adding to it will be even faster than with LinkedList because LinkedList will have to deal with its pointers, while huge ArrayList just sets value at given index.

enter image description here

Check out for more ArrayList and LinkedList differences.

Split a large dataframe into a list of data frames based on common value in column

You can just as easily access each element in the list using e.g. path[[1]]. You can't put a set of matrices into an atomic vector and access each element. A matrix is an atomic vector with dimension attributes. I would use the list structure returned by split, it's what it was designed for. Each list element can hold data of different types and sizes so it's very versatile and you can use *apply functions to further operate on each element in the list. Example below.

#  For reproducibile data
set.seed(1)

#  Make some data
userid <- rep(1:2,times=4)
data1 <- replicate(8 , paste( sample(letters , 3 ) , collapse = "" ) )
data2 <- sample(10,8)
df <- data.frame( userid , data1 , data2 )

#  Split on userid
out <- split( df , f = df$userid )
#$`1`
#  userid data1 data2
#1      1   gjn     3
#3      1   yqp     1
#5      1   rjs     6
#7      1   jtw     5

#$`2`
#  userid data1 data2
#2      2   xfv     4
#4      2   bfe    10
#6      2   mrx     2
#8      2   fqd     9

Access each element using the [[ operator like this:

out[[1]]
#  userid data1 data2
#1      1   gjn     3
#3      1   yqp     1
#5      1   rjs     6
#7      1   jtw     5

Or use an *apply function to do further operations on each list element. For instance, to take the mean of the data2 column you could use sapply like this:

sapply( out , function(x) mean( x$data2 ) )
#   1    2 
#3.75 6.25 

How to display pie chart data values of each slice in chart.js

You can make use of PieceLabel plugin for Chart.js.

{ pieceLabel: { mode: 'percentage', precision: 2 } }

Demo | Documentation

The plugin appears to have a new location (and name): Demo Docs.

How do I open port 22 in OS X 10.6.7

There are 3 solutions available for these.

1) Enable remote login using below command - sudo systemsetup -setremotelogin on

2) In Mac, go to System Preference -> Sharing -> enable Remote Login that's it. 100% working solution

3) Final and most important solution is - Check your private area network connection . Sometime remote login isn't allow inside the local area network.

Kindly try to connect your machine using personal network like mobile network, Hotspot etc.

add controls vertically instead of horizontally using flow layout

I used a BoxLayout and set its second parameter as BoxLayout.Y_AXIS and it worked for me:

panel.setLayout(new BoxLayout(panel, BoxLayout.Y_AXIS));

C#: Looping through lines of multiline string

Here's a quick code snippet that will find the first non-empty line in a string:

string line1;
while (
    ((line1 = sr.ReadLine()) != null) &&
    ((line1 = line1.Trim()).Length == 0)
)
{ /* Do nothing - just trying to find first non-empty line*/ }

if(line1 == null){ /* Error - no non-empty lines in string */ }

Git - remote: Repository not found

In our case it was simply a case of giving write rights in github. Initially the user had only read rights and it was giving this error.

VSCode cannot find module '@angular/core' or any other modules

In my case, when i upgrade vs project to angular 10, I had this errors.

Angular cli creates tsconfig.json, tsconfig.base.json and tsconfig.app.json when i delete tsconfig.json and rename tsconfig.base.json to tsconfig.ts all things will Ok. you must also change extends inside tsconfig.app.json to tsconfig.json

Cannot import keras after installation

I had pip referring by default to pip3, which made me download the libs for python3. On the contrary I launched the shell as python (which opened python 2) and the library wasn't installed there obviously.

Once I matched the names pip3 -> python3, pip -> python (2) all worked.

How to implement zoom effect for image view in android?

Here is one of the most efficient way, it works smoothly:

https://github.com/MikeOrtiz/TouchImageView

Place TouchImageView.java in your project. It can then be used the same as ImageView.

Example:

TouchImageView img = (TouchImageView) findViewById(R.id.img);

If you are using TouchImageView in xml, then you must provide the full package name, because it is a custom view.

Example:

    <com.example.touch.TouchImageView
            android:id="@+id/img”
            android:layout_width="match_parent"
            android:layout_height="match_parent" />

Apply CSS to jQuery Dialog Buttons

Maybe something like this?

$('.ui-state-default:first').addClass('classForCancelButton');

How do I get the unix timestamp in C as an int?

For 32-bit systems:

fprintf(stdout, "%u\n", (unsigned)time(NULL)); 

For 64-bit systems:

fprintf(stdout, "%lu\n", (unsigned long)time(NULL)); 

Get the value of a dropdown in jQuery

Pass the id and hold into a variable and pass the variable where ever you want.

var temp = $('select[name=ID Name]').val();

Android - Adding at least one Activity with an ACTION-VIEW intent-filter after Updating SDK version 23

From official documentation :

To enable Google to crawl your app content and allow users to enter your app from search results, you must add intent filters for the relevant activities in your app manifest. These intent filters allow deep linking to the content in any of your activities. For example, the user might click on a deep link to view a page within a shopping app that describes a product offering that the user is searching for.

Using this link Enabling Deep Links for App Content you'll see how to use it.

And using this Test Your App Indexing Implementation how to test it.

The following XML snippet shows how you might specify an intent filter in your manifest for deep linking.

<activity
    android:name="com.example.android.GizmosActivity"
    android:label="@string/title_gizmos" >
    <intent-filter android:label="@string/filter_title_viewgizmos">
        <action android:name="android.intent.action.VIEW" />
        <category android:name="android.intent.category.DEFAULT" />
        <category android:name="android.intent.category.BROWSABLE" />
        <!-- Accepts URIs that begin with "http://www.example.com/gizmos” -->
        <data android:scheme="http"
              android:host="www.example.com"
              android:pathPrefix="/gizmos" />
        <!-- note that the leading "/" is required for pathPrefix-->
        <!-- Accepts URIs that begin with "example://gizmos” -->
        <data android:scheme="example"
              android:host="gizmos" />

    </intent-filter>
</activity>

To test via Android Debug Bridge

$ adb shell am start
        -W -a android.intent.action.VIEW
        -d <URI> <PACKAGE>

$ adb shell am start
        -W -a android.intent.action.VIEW
        -d "example://gizmos" com.example.android

VBA: Conditional - Is Nothing

Just becuase your class object has no variables does not mean that it is nothing. Declaring and object and creating an object are two different things. Look and see if you are setting/creating the object.

Take for instance the dictionary object - just because it contains no variables does not mean it has not been created.

Sub test()

Dim dict As Object
Set dict = CreateObject("scripting.dictionary")

If Not dict Is Nothing Then
    MsgBox "Dict is something!"  '<--- This shows
Else
    MsgBox "Dict is nothing!"
End If

End Sub

However if you declare an object but never create it, it's nothing.

Sub test()

Dim temp As Object

If Not temp Is Nothing Then
    MsgBox "Temp is something!"
Else
    MsgBox "Temp is nothing!" '<---- This shows
End If

End Sub

read complete file without using loop in java

You can try using Scanner if you are using JDK5 or higher.

Scanner scan = new Scanner(file);  
scan.useDelimiter("\\Z");  
String content = scan.next(); 

Or you can also use Guava

String data = Files.toString(new File("path.txt"), Charsets.UTF8);

Get single row result with Doctrine NativeQuery

I just want one result

implies that you expect only one row to be returned. So either adapt your query, e.g.

SELECT player_id
FROM players p
WHERE CONCAT(p.first_name, ' ', p.last_name) = ?
LIMIT 0, 1

(and then use getSingleResult() as recommended by AdrienBrault) or fetch rows as an array and access the first item:

// ...
$players = $query->getArrayResult();
$myPlayer = $players[0];

git rebase: "error: cannot stat 'file': Permission denied"

An alternate solution rather than closing all apps that might be locking the directory as just about every other answer says to do, would be to use a utility that will unlock the files/directory without closing everything. (I hate needing to restart Visual Studio)

LockHunter is the one that I use: https://lockhunter.com/ There are likely others out there as well, but this one has worked great for me.

Align an element to bottom with flexbox

You can use display: flex to position an element to the bottom, but I do not think you want to use flex in this case, as it will affect all of your elements.

To position an element to the bottom using flex try this:

.container {
  display: flex;
}

.button {
  align-self: flex-end;
}

Your best bet is to set position: absolute to the button and set it to bottom: 0, or you can place the button outside the container and use negative transform: translateY(-100%) to bring it in the container like this:

.content {
    height: 400px;
    background: #000;
    color: #fff;
}
.button {
    transform: translateY(-100%);
    display: inline-block;
}

Check this JSFiddle

UPDATE if exists else INSERT in SQL Server 2008

Many people will suggest you use MERGE, but I caution you against it. By default, it doesn't protect you from concurrency and race conditions any more than multiple statements, but it does introduce other dangers:

http://www.mssqltips.com/sqlservertip/3074/use-caution-with-sql-servers-merge-statement/

Even with this "simpler" syntax available, I still prefer this approach (error handling omitted for brevity):

SET TRANSACTION ISOLATION LEVEL SERIALIZABLE;
BEGIN TRANSACTION;
UPDATE dbo.table SET ... WHERE PK = @PK;
IF @@ROWCOUNT = 0
BEGIN
  INSERT dbo.table(PK, ...) SELECT @PK, ...;
END
COMMIT TRANSACTION;

A lot of folks will suggest this way:

SET TRANSACTION ISOLATION LEVEL SERIALIZABLE;
BEGIN TRANSACTION;
IF EXISTS (SELECT 1 FROM dbo.table WHERE PK = @PK)
BEGIN
  UPDATE ...
END
ELSE
BEGIN
  INSERT ...
END
COMMIT TRANSACTION;

But all this accomplishes is ensuring you may need to read the table twice to locate the row(s) to be updated. In the first sample, you will only ever need to locate the row(s) once. (In both cases, if no rows are found from the initial read, an insert occurs.)

Others will suggest this way:

BEGIN TRY
  INSERT ...
END TRY
BEGIN CATCH
  IF ERROR_NUMBER() = 2627
    UPDATE ...
END CATCH

However, this is problematic if for no other reason than letting SQL Server catch exceptions that you could have prevented in the first place is much more expensive, except in the rare scenario where almost every insert fails. I prove as much here:

Not sure what you think you gain by having a single statement; I don't think you gain anything. MERGE is a single statement but it still has to really perform multiple operations anyway - even though it makes you think it doesn't.

Is there a way to list all resources in AWS

EDIT: This answer is deprecated. Check the other answers.

No,
There is no way to get all resources within your account in one go. Each region is independent and for some services like IAM concept of a region does not exist at all. Although there are API calls available to list down resources and services.
For example:

  • To get list of all available regions for your account:

    output, err := client.DescribeRegions(&ec2.DescribeRegionsInput{})
    

  • To get list of IAM users, roles or group you can use:

    client.GetAccountAuthorizationDetails(&iam.GetAccountAuthorizationDetailsInput{})

    You can find more detail about API calls and their use at: https://docs.aws.amazon.com/sdk-for-go/api/service/iam/

    Above link is only for IAM. Similarly, you can find API for all other resources and services.

  • Angular 4: How to include Bootstrap?

    For bootstrap 4.0.0-alph.6

    npm install [email protected] jquery tether --save
    

    Then after this is done run:

    npm install
    

    Now. Open .angular-cli.json file and import bootstrap, jquery, and tether:

    "styles": [
        "styles.css",
        "../node_modules/bootstrap/dist/css/bootstrap.css"
      ],
      "scripts": [
        "../node_modules/jquery/dist/jquery.js",
        "../node_modules/tether/dist/js/tether.js",
        "../node_modules/bootstrap/dist/js/bootstrap.js"
      ]
    

    And now. You ready to go.

    Detect Route Change with react-router

    I came across this question as I was attempting to focus the ChromeVox screen reader to the top of the "screen" after navigating to a new screen in a React single page app. Basically trying to emulate what would happen if this page was loaded by following a link to a new server-rendered web page.

    This solution doesn't require any listeners, it uses withRouter() and the componentDidUpdate() lifecycle method to trigger a click to focus ChromeVox on the desired element when navigating to a new url path.


    Implementation

    I created a "Screen" component which is wrapped around the react-router switch tag which contains all the apps screens.

    <Screen>
      <Switch>
        ... add <Route> for each screen here...
      </Switch>
    </Screen>
    

    Screen.tsx Component

    Note: This component uses React + TypeScript

    import React from 'react'
    import { RouteComponentProps, withRouter } from 'react-router'
    
    class Screen extends React.Component<RouteComponentProps> {
      public screen = React.createRef<HTMLDivElement>()
      public componentDidUpdate = (prevProps: RouteComponentProps) => {
        if (this.props.location.pathname !== prevProps.location.pathname) {
          // Hack: setTimeout delays click until end of current
          // event loop to ensure new screen has mounted.
          window.setTimeout(() => {
            this.screen.current!.click()
          }, 0)
        }
      }
      public render() {
        return <div ref={this.screen}>{this.props.children}</div>
      }
    }
    
    export default withRouter(Screen)
    
    

    I had tried using focus() instead of click(), but click causes ChromeVox to stop reading whatever it is currently reading and start again where I tell it to start.

    Advanced note: In this solution, the navigation <nav> which inside the Screen component and rendered after the <main> content is visually positioned above the main using css order: -1;. So in pseudo code:

    <Screen style={{ display: 'flex' }}>
      <main>
      <nav style={{ order: -1 }}>
    <Screen>
    

    If you have any thoughts, comments, or tips about this solution, please add a comment.

    Convert character to ASCII code in JavaScript

    JavaScript stores strings as UTF-16 (double byte) so if you want to ignore the second byte just strip it out with a bitwise & operator on 0000000011111111 (ie 255):

    'a'.charCodeAt(0) & 255 === 97; // because 'a' = 97 0 
    'b'.charCodeAt(0) & 255 === 98; // because 'b' = 98 0 
    '?'.charCodeAt(0) & 255 === 19; // because '?' = 19 39
    

    Sum function in VBA

    Range("A1").Function="=SUM(Range(Cells(2,1),Cells(3,2)))"
    

    won't work because worksheet functions (when actually used on a worksheet) don't understand Range or Cell

    Try

    Range("A1").Formula="=SUM(" & Range(Cells(2,1),Cells(3,2)).Address(False,False) & ")"
    

    What is the meaning of Bus: error 10 in C

    this is because str is pointing to a string literal means a constant string ...but you are trying to modify it by copying . Note : if it would have been an error due to memory allocation it would have been given segmentation fault at the run time .But this error is coming due to constant string modification or you can go through the below for more details abt bus error :

    Bus errors are rare nowadays on x86 and occur when your processor cannot even attempt the memory access requested, typically:

    • using a processor instruction with an address that does not satisfy its alignment requirements.

    Segmentation faults occur when accessing memory which does not belong to your process, they are very common and are typically the result of:

    • using a pointer to something that was deallocated.
    • using an uninitialized hence bogus pointer.
    • using a null pointer.
    • overflowing a buffer.

    To be more precise this is not manipulating the pointer itself that will cause issues, it's accessing the memory it points to (dereferencing).

    How to add a 'or' condition in #ifdef

    May use this-

    #if defined CONDITION1 || defined CONDITION2
    //your code here
    #endif
    

    This also does the same-

    #if defined(CONDITION1) || defined(CONDITION2)
    //your code here
    #endif
    

    Further-

    • AND: #if defined CONDITION1 && defined CONDITION2
    • XOR: #if defined CONDITION1 ^ defined CONDITION2
    • AND NOT: #if defined CONDITION1 && !defined CONDITION2

    Converting any string into camel case

    If anyone is using lodash, there is a _.camelCase() function.

    _.camelCase('Foo Bar');
    // ? 'fooBar'
    
    _.camelCase('--foo-bar--');
    // ? 'fooBar'
    
    _.camelCase('__FOO_BAR__');
    // ? 'fooBar'
    

    What's the best visual merge tool for Git?

    My favorite visual merge tool is SourceGear DiffMerge

    • It is free.
    • Cross-platform (Windows, OS X, and Linux).
    • Clean visual UI
    • All diff features you'd expect (Diff, Merge, Folder Diff).
    • Command line interface.
    • Usable keyboard shortcuts.

    User interface

    Convert HTML + CSS to PDF

    Perhaps you might try and use Tidy before handing the file to the converter. If one of the renderer chokes on some HTML problem (like unclosed tag), it might help it.

    Batch - If, ElseIf, Else

    Recommendation. Do not use user-added REM statements to block batch steps. Use conditional GOTO instead. That way you can predefine and test the steps and options. The users also get much simpler changes and better confidence.

    @Echo on
    rem Using flags to control command execution
    
    SET ExecuteSection1=0
    SET ExecuteSection2=1
    
    @echo off
    
    IF %ExecuteSection1%==0 GOTO EndSection1
    ECHO Section 1 Here
    
    :EndSection1
    
    IF %ExecuteSection2%==0 GOTO EndSection2
    ECHO Section 2 Here
    :EndSection2
    

    How to display pandas DataFrame of floats using a format string for columns?

    If you don't want to modify the dataframe, you could use a custom formatter for that column.

    import pandas as pd
    pd.options.display.float_format = '${:,.2f}'.format
    df = pd.DataFrame([123.4567, 234.5678, 345.6789, 456.7890],
                      index=['foo','bar','baz','quux'],
                      columns=['cost'])
    
    
    print df.to_string(formatters={'cost':'${:,.2f}'.format})
    

    yields

            cost
    foo  $123.46
    bar  $234.57
    baz  $345.68
    quux $456.79
    

    How to get date and time from server

    You should set the timezone to the one of the timezones you want.

    // set default timezone
    date_default_timezone_set('America/Chicago'); // CDT
    
    $info = getdate();
    $date = $info['mday'];
    $month = $info['mon'];
    $year = $info['year'];
    $hour = $info['hours'];
    $min = $info['minutes'];
    $sec = $info['seconds'];
    
    $current_date = "$date/$month/$year == $hour:$min:$sec";
    

    Or a much shorter version:

    // set default timezone
    date_default_timezone_set('America/Chicago'); // CDT
    
    $current_date = date('d/m/Y == H:i:s');
    

    How to solve SQL Server Error 1222 i.e Unlock a SQL Server table

    It's been a while, but last time I had something similar:

    ROLLBACK TRAN
    

    or trying to

    COMMIT
    

    what had allready been done free'd everything up so I was able to clear things out and start again.

    What generates the "text file busy" message in Unix?

    One of my experience:

    I always change the default keyboard shortcut of Chrome through reverse engineering. After modification, I forgot to close Chrome and ran the following:

    sudo cp chrome /opt/google/chrome/chrome
    cp: cannot create regular file '/opt/google/chrome/chrome': Text file busy
    

    Using strace, you can find the more details:

    sudo strace cp ./chrome /opt/google/chrome/chrome 2>&1 |grep 'Text file busy'
    open("/opt/google/chrome/chrome", O_WRONLY|O_TRUNC) = -1 ETXTBSY (Text file busy)
    

    convert:not authorized `aaaa` @ error/constitute.c/ReadImage/453

    I use many times the ImageMagic convert command to convert *.tif files to *.pdf files.

    I don't know why but today I began to receive the following error:

    convert: not authorized `a.pdf' @ error/constitute.c/WriteImage/1028.
    

    After issuing the command:

    convert a.tif a.pdf
    

    After reading the above answers I edited the file /etc/ImageMagick-6/policy.xml

    and changed the line:

    policy domain="coder" rights="none" pattern="PDF" 
    

    to

    policy domain="coder" rights="read|write" pattern="PDF"
    

    and now everything works fine.

    I have "ImageMagick 6.8.9-9 Q16 x86_64 2018-09-28" on "Ubuntu 16.04.5 LTS".

    Regex: match word that ends with "Id"

    Try this regular expression:

    \w*Id\b
    

    \w* allows word characters in front of Id and the \b ensures that Id is at the end of the word (\b is word boundary assertion).

    Error parsing yaml file: mapping values are not allowed here

    Maybe this will help someone else, but I've seen this error when the RHS of the mapping contains a colon without enclosing quotes, such as:

    someKey: another key: Change to make today: work out more

    should be

    someKey: another key: "Change to make today: work out more"

    Twitter Bootstrap Form File Element Upload Button

    With no additional plugin required, this bootstrap solution works great for me:

    <div style="position:relative;">
            <a class='btn btn-primary' href='javascript:;'>
                Choose File...
                <input type="file" style='position:absolute;z-index:2;top:0;left:0;filter: alpha(opacity=0);-ms-filter:"progid:DXImageTransform.Microsoft.Alpha(Opacity=0)";opacity:0;background-color:transparent;color:transparent;' name="file_source" size="40"  onchange='$("#upload-file-info").html($(this).val());'>
            </a>
            &nbsp;
            <span class='label label-info' id="upload-file-info"></span>
    </div>
    

    demo:

    http://jsfiddle.net/haisumbhatti/cAXFA/1/ (bootstrap 2)

    enter image description here

    http://jsfiddle.net/haisumbhatti/y3xyU/ (bootstrap 3)

    enter image description here

    How can I toggle word wrap in Visual Studio?

    For Visual Studio 2017 do the following:

    Tools > Options > All Languages, then check or uncheck the checkbox based on your preference. As you can see in below image :

    Firefox Add-on RESTclient - How to input POST parameters?

    You can send the parameters in the URL of the POST request itself.

    Example URL:

    localhost:8080/abc/getDetails?paramter1=value1&parameter2=value2
    

    Once you copy such type of URL in Firefox REST client make a POST call to the server you want

    Is std::vector copying the objects with a push_back?

    Why did it take a lot of valgrind investigation to find this out! Just prove it to yourself with some simple code e.g.

    std::vector<std::string> vec;
    
    {
          std::string obj("hello world");
          vec.push_pack(obj);
    }
    
    std::cout << vec[0] << std::endl;  
    

    If "hello world" is printed, the object must have been copied

    How do I move a file from one location to another in Java?

    You could execute an external tool for that task (like copy in windows environments) but, to keep the code portable, the general approach is to:

    1. read the source file into memory
    2. write the content to a file at the new location
    3. delete the source file

    File#renameTo will work as long as source and target location are on the same volume. Personally I'd avoid using it to move files to different folders.

    How can I find last row that contains data in a specific column?

    Sub test()
        MsgBox Worksheets("sheet_name").Range("A65536").End(xlUp).Row
    End Sub
    

    This is looking for a value in column A because of "A65536".

    maxlength ignored for input type="number" in Chrome

    Chrome (technically, Blink) will not implement maxlength for <input type="number">.

    The HTML5 specification says that maxlength is only applicable to the types text, url, e-mail, search, tel, and password.

    How to remove all subviews of a view in Swift?

    you have to try this

    func clearAllScrollSubView ()
    {
        let theSubviews = itemsScrollView.subviews
    
        for (var view) in theSubviews
        {
    
            if view is UIView
            {
                view.removeFromSuperview()
            }
    
        }
    }
    

    Html.BeginForm and adding properties

    You can also use the following syntax for the strongly typed version:

    <% using (Html.BeginForm<SomeController>(x=> x.SomeAction(), 
              FormMethod.Post, 
              new { enctype = "multipart/form-data" })) 
       { %>
    

    Suppress InsecureRequestWarning: Unverified HTTPS request is being made in Python2.6

    The HTTPS certificate verification security measure isn't something to be discarded light-heartedly. The Man-in-the-middle attack that it prevents safeguards you from a third party e.g. sipping a virus in or tampering with or stealing your data.
    Even if you only intend to do that in a test environment, you can easily forget to undo it when moving elsewhere.

    Instead, read the relevant section on the provided link and do as it says. The way specific for requests (which bundles with its own copy of urllib3), as per CA Certificates — Advanced Usage — Requests 2.8.1 documentation:

    • requests ships with its own certificate bundle (but it can only be updated together with the module)
    • it will use (since requests v2.4.0) the certifi package instead if it's installed
    • In a test environment, you can easily slip a test certificate into certifi as per how do I update root certificates of certifi? . E.g. if you replace its bundle with just your test certificate, you will immediately see it if you forget to undo that when moving to production.

    Finally, with today's government-backed global hacking operations like Tailored Access Operations and the Great Firewall of China that target network infrastructure, falling under a MITM attack is more probable than you think.

    What is the default maximum heap size for Sun's JVM from Java SE 6?

    one way is if you have a jdk installed , in bin folder there is a utility called jconsole(even visualvm can be used). Launch it and connect to the relevant java process and you can see what are the heap size settings set and many other details

    When running headless or cli only, jConsole can be used over lan, if you specify a port to connect on when starting the service in question.

    ORA-01652: unable to extend temp segment by 128 in tablespace SYSTEM: How to extend?

    Each tablespace has one or more datafiles that it uses to store data.

    The max size of a datafile depends on the block size of the database. I believe that, by default, that leaves with you with a max of 32gb per datafile.

    To find out if the actual limit is 32gb, run the following:

    select value from v$parameter where name = 'db_block_size';
    

    Compare the result you get with the first column below, and that will indicate what your max datafile size is.

    I have Oracle Personal Edition 11g r2 and in a default install it had an 8,192 block size (32gb per data file).

    Block Sz   Max Datafile Sz (Gb)   Max DB Sz (Tb)
    
    --------   --------------------   --------------
    
       2,048                  8,192          524,264
    
       4,096                 16,384        1,048,528
    
       8,192                 32,768        2,097,056
    
      16,384                 65,536        4,194,112
    
      32,768                131,072        8,388,224
    

    You can run this query to find what datafiles you have, what tablespaces they are associated with, and what you've currrently set the max file size to (which cannot exceed the aforementioned 32gb):

    select bytes/1024/1024 as mb_size,
           maxbytes/1024/1024 as maxsize_set,
           x.*
    from   dba_data_files x
    

    MAXSIZE_SET is the maximum size you've set the datafile to. Also relevant is whether you've set the AUTOEXTEND option to ON (its name does what it implies).

    If your datafile has a low max size or autoextend is not on you could simply run:

    alter database datafile 'path_to_your_file\that_file.DBF' autoextend on maxsize unlimited;
    

    However if its size is at/near 32gb an autoextend is on, then yes, you do need another datafile for the tablespace:

    alter tablespace system add datafile 'path_to_your_datafiles_folder\name_of_df_you_want.dbf' size 10m autoextend on maxsize unlimited;
    

    What are the benefits of learning Vim?

    The amazing ubiquity of Vim, and the even more amazing ubiquity of Vi-clones in general, on Unix systems alone is enough to make it worth learning.

    Besides that, the whole Vi-style thinking is something that I really think has made me a bit more productive. For a person not used to modes such as the command mode and insert mode, it seems a bit excessive to have to enter a mode just to insert text. But, when one has been using Vim for a few months, and has learned quite a few tips and tricks, Vim seems to be an asset that seems to be worth it.

    Of course, the Emacs crowd says the same thing regarding Emacs-style thinking, but I gave up on learning Emacs because Vim was simpler and did the job for me.

    Styling Password Fields in CSS

    The problem is that (as of 2016), for the password field, Firefox and Internet Explorer use the character "Black Circle" (?), which uses the Unicode code point 25CF, but Chrome uses the character "Bullet" (•), which uses the Unicode code point 2022.

    As you can see, even in the StackOverflow font the two characters have different sizes.

    The font you're using, "Lucida Sans Unicode", has an even greater disparity between the sizes of these two characters, leading to you noticing the difference.

    The simple solution is to use a font in which both characters have similar sizes.

    The fix could thus be to use a default font of the browser, which should render the characters in the password field just fine:

    input[type="password"] {
        font-family: caption;
    }
    

    VBA Excel Provide current Date in Text box

    The easy way to do this is to put the Date function you want to use in a Cell, and link to that cell from the textbox with the LinkedCell property.

    From VBA you might try using:

    textbox.Value = Format(Date(),"mm/dd/yy")
    

    Is it safe to shallow clone with --depth 1, create commits, and pull updates again?

    See some of the answers to my similar question why-cant-i-push-from-a-shallow-clone and the link to the recent thread on the git list.

    Ultimately, the 'depth' measurement isn't consistent between repos, because they measure from their individual HEADs, rather than (a) your Head, or (b) the commit(s) you cloned/fetched, or (c) something else you had in mind.

    The hard bit is getting one's Use Case right (i.e. self-consistent), so that distributed, and therefore probably divergent repos will still work happily together.

    It does look like the checkout --orphan is the right 'set-up' stage, but still lacks clean (i.e. a simple understandable one line command) guidance on the "clone" step. Rather it looks like you have to init a repo, set up a remote tracking branch (you do want the one branch only?), and then fetch that single branch, which feels long winded with more opportunity for mistakes.

    Edit: For the 'clone' step see this answer

    Mod of negative number is melting my brain

    Please note that C# and C++'s % operator is actually NOT a modulo, it's remainder. The formula for modulo that you want, in your case, is:

    float nfmod(float a,float b)
    {
        return a - b * floor(a / b);
    }
    

    You have to recode this in C# (or C++) but this is the way you get modulo and not a remainder.

    Console.WriteLine and generic List

    public static void WriteLine(this List<int> theList)
    {
      foreach (int i in list)
      {
        Console.Write("{0}\t", t.ToString());
      }
      Console.WriteLine();
    }
    

    Then, later...

    list.WriteLine();
    

    Is there an API to get bank transaction and bank balance?

    Also check out the open financial exchange (ofx) http://www.ofx.net/

    This is what apps like quicken, ms money etc use.

    Chmod recursively

    Adding executable permissions, recursively, to all files (not folders) within the current folder with sh extension:

    find . -name '*.sh' -type f | xargs chmod +x

    * Notice the pipe (|)

    Rails Model find where not equal

    The only way you can get it fancier is with MetaWhere.

    MetaWhere has a newer cousin which is called Squeel which allows code like this:

    GroupUser.where{user_id != me}
    

    It goes without saying, that if this is the only refactor you are going to make, it is not worth using a gem and I would just stick with what you got. Squeel is useful in situations where you have many complex queries interacting with Ruby code.

    How can I make my layout scroll both horizontally and vertically?

    Since other solutions are old and either poorly-working or not working at all, I've modified NestedScrollView, which is stable, modern and it has all you expect from a scroll view. Except for horizontal scrolling.

    Here's the repo: https://github.com/ultimate-deej/TwoWayNestedScrollView

    I've made no changes, no "improvements" to the original NestedScrollView expect for what was absolutely necessary. The code is based on androidx.core:core:1.3.0, which is the latest stable version at the time of writing.

    All of the following works:

    • Lift on scroll (since it's basically a NestedScrollView)
    • Edge effects in both dimensions
    • Fill viewport in both dimensions

    'Microsoft.ACE.OLEDB.16.0' provider is not registered on the local machine. (System.Data)

    For anyone that is still stuck on this issue after trying the above. If you are right-clicking on the database and going to tasks->import, then here is the issue. Go to your start menu and under sql server, find the x64 bit import export wizard and try that. Worked like a charm for me, but it took me FAR too long to find it Microsoft!

    How do you run CMD.exe under the Local System Account?

    if you can write a batch file that does not need to be interactive, try running that batch file as a service, to do what needs to be done.

    matplotlib does not show my drawings although I call pyplot.show()

    If I set my backend to template in ~/.matplotlib/matplotlibrc, then I can reproduce your symptoms:

    ~/.matplotlib/matplotlibrc:

    # backend      : GtkAgg
    backend      : template
    

    Note that the file matplotlibrc may not be in directory ~/.matplotlib/. In this case, the following code shows where it is:

    >>> import matplotlib
    >>> matplotlib.matplotlib_fname()
    

    In [1]: import matplotlib.pyplot as p
    
    In [2]: p.plot(range(20),range(20))
    Out[2]: [<matplotlib.lines.Line2D object at 0xa64932c>]
    
    In [3]: p.show()
    

    If you edit ~/.matplotlib/matplotlibrc and change the backend to something like GtkAgg, you should see a plot. You can list all the backends available on your machine with

    import matplotlib.rcsetup as rcsetup
    print(rcsetup.all_backends)
    

    It should return a list like:

    ['GTK', 'GTKAgg', 'GTKCairo', 'FltkAgg', 'MacOSX', 'QtAgg', 'Qt4Agg',
    'TkAgg', 'WX', 'WXAgg', 'CocoaAgg', 'agg', 'cairo', 'emf', 'gdk', 'pdf',
    'ps', 'svg', 'template']
    

    Reference:

    How to make a radio button unchecked by clicking it?

    You could use the checked property of a radio button to uncheck it.

    Something like this:

    <script>
     function uncheck()
     {
      document.getElementById('myRadio').checked = false;        
     }
     function check()
     {
      document.getElementById('myRadio').checked = true;        
     }
    </script>
    <input id="myRadio" type="radio" checked="checked"/>
    <button onclick="uncheck();">Uncheck</button>
    <button onclick="check();">Check</button>
    

    ?See it in action here: http://jsfiddle.net/wgYNa/

    Getting TypeError: __init__() missing 1 required positional argument: 'on_delete' when trying to add parent table after child table with entries

    Since Django 2.0 the ForeignKey field requires two positional arguments:

    1. the model to map to
    2. the on_delete argument
    categorie = models.ForeignKey('Categorie', on_delete=models.PROTECT)
    

    Here are some methods can used in on_delete

    1. CASCADE

    Cascade deletes. Django emulates the behavior of the SQL constraint ON DELETE CASCADE and also deletes the object containing the ForeignKey

    1. PROTECT

    Prevent deletion of the referenced object by raising ProtectedError, a subclass of django.db.IntegrityError.

    1. DO_NOTHING

    Take no action. If your database backend enforces referential integrity, this will cause an IntegrityError unless you manually add an SQL ON DELETE constraint to the database field.

    you can find more about on_delete by reading the documentation.

    What is the preferred Bash shebang?

    It really depends on how you write your bash scripts. If your /bin/sh is symlinked to bash, when bash is invoked as sh, some features are unavailable.

    If you want bash-specific, non-POSIX features, use #!/bin/bash

    How to escape the % (percent) sign in C's printf?

    You can use %%:

    printf("100%%");
    

    The result is:

    100%

    What does "for" attribute do in HTML <label> tag?

    The <label> tag allows you to click on the label, and it will be treated like clicking on the associated input element. There are two ways to create this association:

    One way is to wrap the label element around the input element:

    <label>Input here:
        <input type='text' name='theinput' id='theinput'>
    </label>
    

    The other way is to use the for attribute, giving it the ID of the associated input:

    <label for="theinput">Input here:</label>
    <input type='text' name='whatever' id='theinput'>
    

    This is especially useful for use with checkboxes and buttons, since it means you can check the box by clicking on the associated text instead of having to hit the box itself.

    Read more about this element in MDN.

    How to make php display \t \n as tab and new line instead of characters

    Put it in double quotes:

    echo "\t";
    

    Single quotes do not expand escaped characters.

    Use the documentation when in doubt.

    Generating a list of pages (not posts) without the index file

    I have never used jekyll, but it's main page says that it uses Liquid, and according to their docs, I think the following should work:

    <ul> {% for page in site.pages %}     {% if page.title != 'index' %}     <li><div class="drvce"><a href="{{ page.url }}">{{ page.title }}</a></div></li>     {% endif %} {% endfor %} </ul> 

    How to calculate the time interval between two time strings

    I like how this guy does it — https://amalgjose.com/2015/02/19/python-code-for-calculating-the-difference-between-two-time-stamps. Not sure if it has some cons.

    But looks neat for me :)

    from datetime import datetime
    from dateutil.relativedelta import relativedelta
    
    t_a = datetime.now()
    t_b = datetime.now()
    
    def diff(t_a, t_b):
        t_diff = relativedelta(t_b, t_a)  # later/end time comes first!
        return '{h}h {m}m {s}s'.format(h=t_diff.hours, m=t_diff.minutes, s=t_diff.seconds)
    

    Regarding to the question you still need to use datetime.strptime() as others said earlier.

    How does Junit @Rule work?

    Rules are used to add additional functionality which applies to all tests within a test class, but in a more generic way.

    For instance, ExternalResource executes code before and after a test method, without having to use @Before and @After. Using an ExternalResource rather than @Before and @After gives opportunities for better code reuse; the same rule can be used from two different test classes.

    The design was based upon: Interceptors in JUnit

    For more information see JUnit wiki : Rules.

    How to add bootstrap in angular 6 project?

    using command

    npm install bootstrap --save
    

    open .angular.json old (.angular-cli.json ) file find the "styles" add the bootstrap css file

    "styles": [
           "src/styles.scss",
           "node_modules/bootstrap/dist/css/bootstrap.min.css"
    ],
    

    How to Retrieve value from JTextField in Java Swing?

    import java.awt.*;
    import java.awt.event.*;
    import javax.swing.*;
    
    public class Swingtest extends JFrame implements ActionListener
    {
        JTextField txtdata;
        JButton calbtn = new JButton("Calculate");
    
        public Swingtest()
        {
            JPanel myPanel = new JPanel();
            add(myPanel);
            myPanel.setLayout(new GridLayout(3, 2));
            myPanel.add(calbtn);
            calbtn.addActionListener(this);
            txtdata = new JTextField();
            myPanel.add(txtdata);
        }
    
        public void actionPerformed(ActionEvent e)
        {
            if (e.getSource() == calbtn) {
                String data = txtdata.getText(); //perform your operation
                System.out.println(data);
            }
        }
    
        public static void main(String args[])
        {
            Swingtest g = new Swingtest();
            g.setLocation(10, 10);
            g.setSize(300, 300);
            g.setVisible(true);
        }
    }
    

    now its working

    how to make negative numbers into positive

    Well, in mathematics to convert a negative number to a positive number you just need to multiple the negative number by -1;

    Then your solution could be like this:

    a = a * -1;
    

    or shorter:

    a *= -1;
    

    array filter in python?

    >>> a = set([6, 7, 8, 9, 10, 11, 12])
    >>> sub_a = set([6, 9, 12])
    >>> a - sub_a
    set([8, 10, 11, 7])
    

    Best practices for Storyboard login screen, handling clearing of data upon logout

    Here is what I ended up doing to accomplish everything. The only thing you need to consider in addition to this is (a) the login process and (b) where you are storing your app data (in this case, I used a singleton).

    Storyboard showing login view controller and main tab controller

    As you can see, the root view controller is my Main Tab Controller. I did this because after the user has logged in, I want the app to launch directly to the first tab. (This avoids any "flicker" where the login view shows temporarily.)

    AppDelegate.m

    In this file, I check whether the user is already logged in. If not, I push the login view controller. I also handle the logout process, where I clear data and show the login view.

    - (BOOL)application:(UIApplication *)application didFinishLaunchingWithOptions:(NSDictionary *)launchOptions
    {
    
        // Show login view if not logged in already
        if(![AppData isLoggedIn]) {
            [self showLoginScreen:NO];
        }
    
        return YES;
    }
    
    -(void) showLoginScreen:(BOOL)animated
    {
    
        // Get login screen from storyboard and present it
        UIStoryboard *storyboard = [UIStoryboard storyboardWithName:@"MainStoryboard" bundle:nil];
        LoginViewController *viewController = (LoginViewController *)[storyboard instantiateViewControllerWithIdentifier:@"loginScreen"];
        [self.window makeKeyAndVisible];
        [self.window.rootViewController presentViewController:viewController
                                                 animated:animated
                                               completion:nil];
    }
    
    -(void) logout
    {
        // Remove data from singleton (where all my app data is stored)
        [AppData clearData];
    
       // Reset view controller (this will quickly clear all the views)
       UIStoryboard *storyboard = [UIStoryboard storyboardWithName:@"MainStoryboard" bundle:nil];
       MainTabControllerViewController *viewController = (MainTabControllerViewController *)[storyboard instantiateViewControllerWithIdentifier:@"mainView"];
       [self.window setRootViewController:viewController];
    
       // Show login screen
       [self showLoginScreen:NO];
    
    }
    

    LoginViewController.m

    Here, if the login is successful, I simply dismiss the view and send a notification.

    -(void) loginWasSuccessful
    {
    
         // Send notification
         [[NSNotificationCenter defaultCenter] postNotificationName:@"loginSuccessful" object:self];
    
         // Dismiss login screen
         [self dismissViewControllerAnimated:YES completion:nil];
    
    }
    

    How to replace specific values in a oracle database column?

    In Oracle, there is the concept of schema name, so try using this

    update schemname.tablename t
    set t.columnname = replace(t.columnname, t.oldvalue, t.newvalue);
    

    Switch case in C# - a constant value is expected

    You can only match to constants in switch statements.


    Example:

    switch (variable1)
    {
        case 1: // A hard-coded value
            // Code
            break;
        default:
            // Code
            break;
    }
    

    Successful!


    switch (variable1)
    {
        case variable2:
            // Code
            break;
        default:
            // Code
            break;
    }
    

    CS0150 A constant value is expected.

    JPA & Criteria API - Select only specific columns

    cq.select(cb.construct(entityClazz.class, root.get("ID"), root.get("VERSION")));  // HERE IS NO ERROR
    

    https://wiki.eclipse.org/EclipseLink/UserGuide/JPA/Basic_JPA_Development/Querying/Criteria#Constructors

    Excel is not updating cells, options > formula > workbook calculation set to automatic

    Add this to your macro and it will recalculate all the cells and formulae.

    Call Application.CalculateFullRebuild
    

    Hope it has been already fixed.

    PS The above code is for the people looking for a macro to solve the issue.

    Reference: https://msdn.microsoft.com/en-us/vba/excel-vba/articles/application-calculatefullrebuild-method-excel/

    How to save RecyclerView's scroll position using RecyclerView.State?

    Beginning from version 1.2.0-alpha02 of androidx recyclerView library, it is now automatically managed. Just add it with:

    implementation "androidx.recyclerview:recyclerview:1.2.0-alpha02"
    

    And use:

    adapter.stateRestorationPolicy = StateRestorationPolicy.PREVENT_WHEN_EMPTY
    

    The StateRestorationPolicy enum has 3 options:

    • ALLOW — the default state, that restores the RecyclerView state immediately, in the next layout pass
    • PREVENT_WHEN_EMPTY — restores the RecyclerView state only when the adapter is not empty (adapter.getItemCount() > 0). If your data is loaded async, the RecyclerView waits until data is loaded and only then the state is restored. If you have default items, like headers or load progress indicators as part of your Adapter, then you should use the PREVENT option, unless the default items are added using MergeAdapter. MergeAdapter waits for all of its adapters to be ready and only then it restores the state.
    • PREVENT — all state restoration is deferred until you set ALLOW or PREVENT_WHEN_EMPTY.

    Note that at the time of this answer, recyclerView library is still in alpha03, but alpha phase is not suitable for production purposes.

    How do I import a namespace in Razor View Page?

    I think in order import namespace in razor view, you just need to add below way:

    @using XX.YY.ZZ
    

    File content into unix variable with newlines

    Just if someone is interested in another option:

    content=( $(cat test.txt) )
    
    a=0
    while [ $a -le ${#content[@]} ]
    do
            echo ${content[$a]}
            a=$[a+1]
    done
    

    node.js: read a text file into an array. (Each line an item in the array.)

    file.lines with JFile package

    Pseudo

    var JFile=require('jfile');
    
    var myF=new JFile("./data.txt");
    myF.lines // ["first line","second line"] ....
    

    Don't forget before :

    npm install jfile --save
    

    How do I specify "close existing connections" in sql script

    try this C# code to drop your database

    public static void DropDatabases(string dataBase) {

            string sql =  "ALTER DATABASE "  + dataBase + "SET SINGLE_USER WITH ROLLBACK IMMEDIATE" ;
    
            using (System.Data.SqlClient.SqlConnection connection = new System.Data.SqlClient.SqlConnection(ConfigurationManager.ConnectionStrings["DBRestore"].ConnectionString))
            {
                connection.Open();
                using (System.Data.SqlClient.SqlCommand command = new System.Data.SqlClient.SqlCommand(sql, connection))
                {
                    command.CommandType = CommandType.Text;
                    command.CommandTimeout = 7200;
                    command.ExecuteNonQuery();
                }
                sql = "DROP DATABASE " + dataBase;
                using (System.Data.SqlClient.SqlCommand command = new System.Data.SqlClient.SqlCommand(sql, connection))
                {
                    command.CommandType = CommandType.Text;
                    command.CommandTimeout = 7200;
                    command.ExecuteNonQuery();
                }
            }
        }
    

    Using Position Relative/Absolute within a TD?

    also works if you do a "display: block;" on the td, destroying the td identity, but works!

    How to search for a string in cell array in MATLAB?

    The strcmp and strcmpi functions are the most direct way to do this. They search through arrays.

    strs = {'HA' 'KU' 'LA' 'MA' 'TATA'}
    ix = find(strcmp(strs, 'KU'))
    

    How to update data in one table from corresponding data in another table in SQL Server 2005

     UPDATE Employee SET Empid=emp3.empid 
     FROM EMP_Employee AS emp3
     WHERE Employee.Empid=emp3.empid
    

    Dynamically generating a QR code with PHP

    It's worth adding that, in addition to the QR codes library posted by @abaumg, Google provides a QR Codes API QR Codes APImany thanks to @Toukakoukan for the link update.

    To use this , basically:

    https://chart.googleapis.com/chart?chs=300x300&cht=qr&chl=http%3A%2F%2Fwww.google.com%2F&choe=UTF-8
    
    • 300x300 is the size of the QR image you want to generate,
    • the chl is the url-encoded string you want to change into a QR code, and
    • the choe is the (optional) encoding.

    The link, above, gives more detail, but to use it just have the src of an image point to the manipulated value, like so:

    <img src="https://chart.googleapis.com/chart?chs=300x300&cht=qr&chl=http%3A%2F%2Fwww.google.com%2F&choe=UTF-8" title="Link to Google.com" />
    

    Demo:

    Creating and returning Observable from Angular 2 Service

    Notice that you're using Observable#map to convert the raw Response object your base Observable emits to a parsed representation of the JSON response.

    If I understood you correctly, you want to map again. But this time, converting that raw JSON to instances of your Model. So you would do something like:

    http.get('api/people.json')
      .map(res => res.json())
      .map(peopleData => peopleData.map(personData => new Person(personData)))
    

    So, you started with an Observable that emits a Response object, turned that into an observable that emits an object of the parsed JSON of that response, and then turned that into yet another observable that turned that raw JSON into an array of your models.

    Setting Camera Parameters in OpenCV/Python

    To avoid using integer values to identify the VideoCapture properties, one can use, e.g., cv2.cv.CV_CAP_PROP_FPS in OpenCV 2.4 and cv2.CAP_PROP_FPS in OpenCV 3.0. (See also Stefan's comment below.)

    Here a utility function that works for both OpenCV 2.4 and 3.0:

    # returns OpenCV VideoCapture property id given, e.g., "FPS"
    def capPropId(prop):
      return getattr(cv2 if OPCV3 else cv2.cv,
        ("" if OPCV3 else "CV_") + "CAP_PROP_" + prop)
    

    OPCV3 is set earlier in my utilities code like this:

    from pkg_resources import parse_version
    OPCV3 = parse_version(cv2.__version__) >= parse_version('3')
    

    Laravel: Get base url

    Check this -

    <a href="{{url('/abc/xyz')}}">Go</a>
    

    This is working for me and I hope it will work for you.

    Uncompress tar.gz file

    Try this:

    tar -zxvf file.tar.gz
    

    How to send an email with Gmail as provider using Python?

    great answer from @David, here is for Python 3 without the generic try-except:

    def send_email(user, password, recipient, subject, body):
    
        gmail_user = user
        gmail_pwd = password
        FROM = user
        TO = recipient if type(recipient) is list else [recipient]
        SUBJECT = subject
        TEXT = body
    
        # Prepare actual message
        message = """From: %s\nTo: %s\nSubject: %s\n\n%s
        """ % (FROM, ", ".join(TO), SUBJECT, TEXT)
    
        server = smtplib.SMTP("smtp.gmail.com", 587)
        server.ehlo()
        server.starttls()
        server.login(gmail_user, gmail_pwd)
        server.sendmail(FROM, TO, message)
        server.close()
    

    mysql -> insert into tbl (select from another table) and some default values

    If you want to insert all the columns then

    insert into def select * from abc;
    

    here the number of columns in def should be equal to abc.

    if you want to insert the subsets of columns then

    insert into def (col1,col2, col3 ) select scol1,scol2,scol3 from abc; 
    

    if you want to insert some hardcorded values then

    insert into def (col1, col2,col3) select 'hardcoded value',scol2, scol3 from abc;